Cardiology Flashcards

1
Q

The single most important bedside measurement from which to estimate the volume status

A

JVP

How well did you know this?
1
Not at all
2
3
4
5
Perfectly
2
Q

wave that occurs with atrioventricular (AV) dissociation and right atrial contraction against a closed tricuspid valve

A

cannon a wave

a wave is not present in AF

How well did you know this?
1
Not at all
2
3
4
5
Perfectly
3
Q

waves that are present in progressive TR

A

C-V waves

How well did you know this?
1
Not at all
2
3
4
5
Perfectly
4
Q

Abdominojugular reflux is produced with firm and consistent pressure over the upper portion of the abdomen, preferably over the right upper quadrant, for >___ s.

A positive response is defined by a sustained rise of > __ cm in the JVP during the application of firm abdominal pressure

A

15s
>3 cm

indicates presence of >15 pulmonary artery wedge pressure

How well did you know this?
1
Not at all
2
3
4
5
Perfectly
5
Q

Blood pressure should be measured in both arms, and the difference should be < __mmHg

A

10

How well did you know this?
1
Not at all
2
3
4
5
Perfectly
6
Q

Systolic leg pressures are usually as much as ___ mmHg higher than systolic arm pressures.

A

20

How well did you know this?
1
Not at all
2
3
4
5
Perfectly
7
Q

A weak and delayed pulse (pulsus parvus et tardus) defines what dse ?

A

severe aortic stenosis (AS)

How well did you know this?
1
Not at all
2
3
4
5
Perfectly
8
Q

Abnormal pulse oximetry (a >__% difference between finger and toe oxygen saturation) can be used to detect lower extremity peripheral arterial disease and is comparable in its performance characteristics to the ankle-brachial index.

A

2

How well did you know this?
1
Not at all
2
3
4
5
Perfectly
9
Q

Reversed or paradoxical splitting refers to a pathologic delay in aortic valve closure, such as that which occurs in patients with ______ (5)

A

left bundle branch block, right ventricular pacing, severe AS, HOCM, and acute myocardial ischemia

How well did you know this?
1
Not at all
2
3
4
5
Perfectly
10
Q

With standing, most murmurs diminish, with two exceptions being the murmur of , which becomes louder ___, and that of ___, which lengthens and often is intensified.

A

HOCM

MVP

How well did you know this?
1
Not at all
2
3
4
5
Perfectly
11
Q

Classic triad of findings ECG for pericardial effusion with cardiac tamponade

A

(1) sinus tachycardia
(2) low QRS voltages
(3) electrical alternans

How well did you know this?
1
Not at all
2
3
4
5
Perfectly
12
Q

In CTA, Coronary what calcium scores are considered
moderate ?
severe ?

A

moderate (100–400)
severe (>400).

How well did you know this?
1
Not at all
2
3
4
5
Perfectly
13
Q

considered the most accurate noninvasive technique to evaluate the structure and ejection fraction of the right ventricle

A

CMR

How well did you know this?
1
Not at all
2
3
4
5
Perfectly
14
Q

most common congenital anomaly of the heart

A

bicuspid aortic valve

How well did you know this?
1
Not at all
2
3
4
5
Perfectly
15
Q

An important predictor of outcome in patients with mitral regurgitation of any cause.

A

Ventricular dilatation

in a patient with significant mitral regurgitation, a large portion of the blood being ejected from the left ventricle with every beat is regurgitant, thus artificially increasing the ejection fraction.
Thus, an ejection fraction of 55% in a patient with severe mitral regurgitation may actually represent substantial reduction in myocardial systolic function

How well did you know this?
1
Not at all
2
3
4
5
Perfectly
16
Q

Definition of cardiotoxicity

A

The accepted standard for clinical diagnosis of cardiotoxicity is defined as a >5% reduction in LVEF to <55% in symptomatic patients or a 10% drop in LVEF to <55% in patients who are asymptomatic.

How well did you know this?
1
Not at all
2
3
4
5
Perfectly
17
Q

Most common type of ASD

A

Secundum

occur most commonly in the region of the fossa ovalis

How well did you know this?
1
Not at all
2
3
4
5
Perfectly
18
Q

Warfarin is held starting ___ days prior to the cardiac catheterization to allow the international normalized ratio (INR) to fall to ___ and limit access-site bleeding complications.

A

2-3
<1.7

How well did you know this?
1
Not at all
2
3
4
5
Perfectly
19
Q

What sign is demonstrated by an increase in the left ventricular–aorta pressure gradient with a simultaneous decrease in the aortic pulse pressure following a premature ventricular contraction?

A

Hypertrophic obstructive cardiomyopathy is confirmed by the Brockenbrough-Braunwald sign

How well did you know this?
1
Not at all
2
3
4
5
Perfectly
20
Q

What disease condition/s is/are square root sign seen?

A

Constrictive pericarditis, restrictive cardiomyopathy

How well did you know this?
1
Not at all
2
3
4
5
Perfectly
21
Q

Cardiomyopathy that has amarked increase in right ventricular and pulmonary artery systolic pressures (usually >60 mmHg),

A

Restrictive

How well did you know this?
1
Not at all
2
3
4
5
Perfectly
22
Q

What disease condition has prominent x and y descent?

A

Constrictive pericarditis

How well did you know this?
1
Not at all
2
3
4
5
Perfectly
23
Q

A coronary stenosis of __ % is considered significant

A

50%

How well did you know this?
1
Not at all
2
3
4
5
Perfectly
24
Q

A sinus rate of < ___ beats/min in the awake state in the absence of physical conditioning generally is considered abnormal

A

40

How well did you know this?
1
Not at all
2
3
4
5
Perfectly
25
Q

Treatment options for POTS

A

Volume expansion with salt supplementation, oral fludrocortisone, compression stockings, and the α-agonist midodrine, often in combination, can be helpful. Exercise training has also been purported to improve symptoms.

How well did you know this?
1
Not at all
2
3
4
5
Perfectly
26
Q

Most likely mechanism of MAT

A

triggered automaticity

How well did you know this?
1
Not at all
2
3
4
5
Perfectly
27
Q

The risk of developing VT is greatest in _____ of acute MI.

A

the first hour

How well did you know this?
1
Not at all
2
3
4
5
Perfectly
28
Q

The cardinal symptoms of HF are ____ and ___

A

fatigue and shortness of breath

How well did you know this?
1
Not at all
2
3
4
5
Perfectly
29
Q

Most impt mechanism of dyspnea in HF

A

The most important mechanism is pulmonary congestion with accumulation of interstitial or intra-alveolar fluid, which activates juxtacapillary J receptors, which in turn stimulate the rapid, shallow breathing characteristic of cardiac dyspnea.

How well did you know this?
1
Not at all
2
3
4
5
Perfectly
30
Q

A normal ECG in HF virtually excludes _____

A

LV systolic dysfunction

How well did you know this?
1
Not at all
2
3
4
5
Perfectly
31
Q

For HFreF beta blockers should ideally be restricted to

A

carvedilol, bisoprolol, and metoprolol succinate

How well did you know this?
1
Not at all
2
3
4
5
Perfectly
32
Q

The single most important association of extent of dyssynchrony is ______ on ECG

A

a widened QRS interval on the surface electrocardiogram, particularly in the presence of a left bundle branch block pattern

How well did you know this?
1
Not at all
2
3
4
5
Perfectly
33
Q

Cardiomyopathy that has EF > 60%

A

Hypertrophic

How well did you know this?
1
Not at all
2
3
4
5
Perfectly
34
Q

cardiomyopathy that has LV dimension >60 mm

A

Dilated

How well did you know this?
1
Not at all
2
3
4
5
Perfectly
35
Q

Cardiomyopathy related to valve septum interaction

A

Hypertrophic

How well did you know this?
1
Not at all
2
3
4
5
Perfectly
36
Q

Cardiomyopathy related to endocardial involvement

A

Restrictive

How well did you know this?
1
Not at all
2
3
4
5
Perfectly
37
Q

The most commonly recognized genetic causes of DCM are truncating mutations of the ___

A

ogiant protein titin, encoded by TTN, which maintains sarcomere structure and acts as a key signaling molecule

How well did you know this?
1
Not at all
2
3
4
5
Perfectly
38
Q

Define possible subclinical, probable and definite myocarditis

A

(1) Possible subclinical acute myocarditis is diagnosed when a typical viral syndrome occurs without cardiac symptoms, but with elevated biomarkers of cardiac injury, ECG suggestive of acute injury, reduced left ventricular ejection fraction or regional wall motion abnormality.

(2) Probable acute myocarditis is diagnosed when the above criteria are met and accompanied by cardiac symptoms, such as shortness of breath or chest pain, which can result from pericarditis or myocarditis.

(3) Definite myocarditis is diagnosed when there is histologic or immunohistologic evidence of inflammation on endomyocardial biopsy and does not require any other laboratory or clinical criteria.

How well did you know this?
1
Not at all
2
3
4
5
Perfectly
39
Q

most common infective cause of cardiomyopathy

A

Chaga’s dse

How well did you know this?
1
Not at all
2
3
4
5
Perfectly
40
Q

The most common cause NONinfective inflammation of the heart is

A

granulomatous myocarditis, including both sarcoidosis and giant cell myocarditis.

How well did you know this?
1
Not at all
2
3
4
5
Perfectly
41
Q

most common toxin implicated in chronic DCM

A

Alcohol

Estimates of the alcohol intake necessary to cause cardiomyopathy have been 4–5 ounces or 80–100 g of pure ethanol daily for 5–10 years, about 1 L of wine, 8 beers, or ½ pint of hard liquor. Frequent binge drinking may also be sufficient

How well did you know this?
1
Not at all
2
3
4
5
Perfectly
42
Q

The most common current reason for thyroid abnormalities in the cardiac population is the ______

A

treatment of tachyarrhythmias with amiodarone, a drug with substantial iodine content

How well did you know this?
1
Not at all
2
3
4
5
Perfectly
43
Q

Treatment for HOCM

A

β-Adrenergic blocking agents and L-type calcium channel blockers slow AV nodal conduction and improve symptoms

cardiac glycosides should be avoided, as they may increase contractility and worsen obstruction

How well did you know this?
1
Not at all
2
3
4
5
Perfectly
44
Q

Major risk factors for SCD in HOCM

A

see table

How well did you know this?
1
Not at all
2
3
4
5
Perfectly
45
Q

T-wave changes are more common but are less specific signs of ischemia, unless they are new and deep T-wave inversions ( ____ mV).

A

≥0.3

How well did you know this?
1
Not at all
2
3
4
5
Perfectly
46
Q

Thrombolysis in Myocardial Infarction (TIMI) Trials, which includes seven independent risk factors which include

A

age ≥ 65 years
3 or more of the traditional risk factors for coronary heart disease
known history of coronary artery disease or coronary stenosis of at least 50%
daily aspirin use in the prior week
more than one anginal episode in the past 24 h
ST segment deviation of at least 0.5 mm
elevated cardiac specific biomarker above the upper limit of normal

How well did you know this?
1
Not at all
2
3
4
5
Perfectly
47
Q

The only absolute contraindications to the use of nitrates are _____ and ____

A

hypotension or the recent use of a phosphodiesterase type 5 (PDE-5) inhibitor, sildenafil or vardenafil (within 24 h), or tadalafil (within 48 h).

How well did you know this?
1
Not at all
2
3
4
5
Perfectly
48
Q

Target HR for NSTEMI

A

50-60

How well did you know this?
1
Not at all
2
3
4
5
Perfectly
49
Q

DAPT should continue for at least ____ in patients with NSTEACS, especially those with a drug-eluting stent, to prevent stent thrombosis.

A

1 year

How well did you know this?
1
Not at all
2
3
4
5
Perfectly
49
Q

When is immediate invasive strategy indicated in NSTEMI

A

see table

How well did you know this?
1
Not at all
2
3
4
5
Perfectly
50
Q

Cut off GRACE score to qualify for early invasive strategy in NSTEMI

A

> 140

How well did you know this?
1
Not at all
2
3
4
5
Perfectly
51
Q

Treatment for Prinzmetal angina

A

Nitrates and calcium channel blockers

Aspirin may actually increase the severity of ischemic episodes, possibly as a result of the sensitivity of coronary tone to modest changes in the synthesis of prostacyclin.

Statin therapy has been shown to reduce the risk of major adverse events, although the precise mechanism is not established

How well did you know this?
1
Not at all
2
3
4
5
Perfectly
52
Q

The pain of STEMI may radiate as high as the _____ but not below the ____.

A

as high as the occipital area but not below the umbilicus.

How well did you know this?
1
Not at all
2
3
4
5
Perfectly
53
Q

in STEMI, The nonspecific reaction to myocardial injury is associated with polymorphonuclear leukocytosis, which appears within a few hours after the onset of pain and persists for ___ days; the white blood cell count often reaches levels of 12,000–15,000/μL.

A

3–7

How well did you know this?
1
Not at all
2
3
4
5
Perfectly
54
Q

How do you differentiate Type 1 -5 MI

A

see table

How well did you know this?
1
Not at all
2
3
4
5
Perfectly
55
Q

An idiosyncratic reaction to nitrates, consisting of sudden marked hypotension, sometimes occurs but can be reversed promptly by the rapid administration of intravenous _____

A

atropine

How well did you know this?
1
Not at all
2
3
4
5
Perfectly
56
Q

FMC - device time

A

<=90 mins if PCI not capable

***but if capable, 60 mins (not from garrisons)

How well did you know this?
1
Not at all
2
3
4
5
Perfectly
57
Q

Transfer to PCI capable hospital should be done in ____ mins

A

120min

How well did you know this?
1
Not at all
2
3
4
5
Perfectly
58
Q

Administration of fibrinolytics should be done in ____ mins if transfer to PCI capable hospital would take > 120 mins

A

30m

How well did you know this?
1
Not at all
2
3
4
5
Perfectly
59
Q

Clear contraindications to the use of fibrinolytic agents include

A

a history of cerebrovascular hemorrhage at any time

a nonhemorrhagic stroke or other cerebrovascular event within the past year

marked hypertension (a reliably determined systolic arterial pressure >180 mmHg and/or a diastolic pressure >110 mmHg) at any time during the acute presentation

suspicion of aortic dissection

active internal bleeding (excluding menses)

How well did you know this?
1
Not at all
2
3
4
5
Perfectly
60
Q

Because of the risk of emesis and aspiration soon after STEMI, patients should receive either nothing or only clear liquids by mouth for the first ___ h.

A

4–12

How well did you know this?
1
Not at all
2
3
4
5
Perfectly
61
Q

patients with STEMI should be kept at bed rest for the first

A

6–12 h.

How well did you know this?
1
Not at all
2
3
4
5
Perfectly
62
Q

patients with STEMI resume an upright posture by dangling their feet over the side of the bed and sitting in a chair within .

A

the first 24 h

How well did you know this?
1
Not at all
2
3
4
5
Perfectly
63
Q

for patients with STEMI, by the _____ day, patients typically are ambulating in their room with increasing duration and frequency, and they may shower or stand at the sink to bathe.

A

second or third

How well did you know this?
1
Not at all
2
3
4
5
Perfectly
64
Q

The typical coronary care unit diet for STEMI px should provide ___ of total calories as fat and have a cholesterol content of ≤300 mg/d. Complex carbohydrates should make up ____ of total calories.

A

≤30%
50–55%

How well did you know this?
1
Not at all
2
3
4
5
Perfectly
64
Q

for patients with STEMI, By day 3 after infarction, patients should be increasing their ambulation progressively to a goal of ___ m at least three times a day.

A

185 m (600 ft)

How well did you know this?
1
Not at all
2
3
4
5
Perfectly
65
Q

primary cause of in-hospital death from STEMI

A

Pump failure

How well did you know this?
1
Not at all
2
3
4
5
Perfectly
66
Q

Most patients who had STEMI will be able to return to work within ___ weeks.

A

2–4

How well did you know this?
1
Not at all
2
3
4
5
Perfectly
67
Q

For STEMI px normal sexual activity may be resumed during this period.

A

1-2w

How well did you know this?
1
Not at all
2
3
4
5
Perfectly
68
Q

How do you define angiographic success in PCI

A

A successful procedure (angiographic success), defined as a reduction of the stenosis to less than a 20% diameter narrowing, occurs in 95–99% of patients.

How well did you know this?
1
Not at all
2
3
4
5
Perfectly
69
Q

most common complication of angioplasty

A

restenosis

occurs in 20–50% of patients with balloon angioplasty alone, 10–30% of patients with bare metal stents, and 5–15% of patients with drug-eluting stents within the first year.

How well did you know this?
1
Not at all
2
3
4
5
Perfectly
70
Q

The primary reason for being considered inoperable with CABG is

A

the presence of severe comorbidities such as advanced age, frailty, severe chronic obstructive pulmonary disease (COPD), poor left ventricular function, or lack of suitable surgical conduits or poor distal targets for bypass

How well did you know this?
1
Not at all
2
3
4
5
Perfectly
71
Q

three cardinal symptoms of AS

A

Exertional dyspnea, angina pectoris, and syncope

How well did you know this?
1
Not at all
2
3
4
5
Perfectly
72
Q

Describe the murmur of AS

A

The murmur of AS is characteristically an ejection (mid) systolic murmur that commences shortly after the S1 , increases in intensity to reach a peak toward the middle of ejection, and ends just before aortic valve closure.

How well did you know this?
1
Not at all
2
3
4
5
Perfectly
73
Q

In AS, average time to death after the onset of various symptoms was as follows
angina pectoris, ___
syncope, ____
dyspnea ____

A

angina pectoris, 3 years;
syncope, 3 years;
dyspnea, 2 years;

How well did you know this?
1
Not at all
2
3
4
5
Perfectly
74
Q

in AR A rapidly rising “water-hammer” pulse, which collapses suddenly as arterial pressure falls rapidly during late systole and diastole is called ______

A

(Corrigan’s pulse)

How well did you know this?
1
Not at all
2
3
4
5
Perfectly
75
Q

In AR, capillary pulsations, an alternate flushing and paling of the skin at the root of the nail while pressure is applied to the tip of the nail is called ______

A

(Quincke’s pulse)

How well did you know this?
1
Not at all
2
3
4
5
Perfectly
76
Q

in AR, A booming “pistol-shot” sound can be heard over the femoral arteries is called _____

A

(Traube’s sign)

How well did you know this?
1
Not at all
2
3
4
5
Perfectly
77
Q

in AR to-and-fro murmur that is audible if femoral artery is lightly compressed with steth is called _____

A

(Duroziez’s sign)

How well did you know this?
1
Not at all
2
3
4
5
Perfectly
78
Q

Leading cause of MS

A

Rheumatic fever

How well did you know this?
1
Not at all
2
3
4
5
Perfectly
79
Q

With severe pulmonary hypertension, a pansystolic murmur produced by functional TR may be audible along the left sternal border. This murmur is usually louder during inspiration and diminishes during forced expiration and is called _____

A

(Carvallo’s sign)

How well did you know this?
1
Not at all
2
3
4
5
Perfectly
80
Q

most characteristic auscultatory finding in chronic severe MR

A

systolic murmur of at least grade III/VI intensity

It is usually holosystolic , but as previously noted, it is decrescendo and ceases in mid-to-late systole in patients with acute severe MR.

The systolic murmur of chronic MR is usually most prominent at the apex and radiates to the axilla

How well did you know this?
1
Not at all
2
3
4
5
Perfectly
81
Q

most common abnormality leading to primary mitral regurgitation

A

MVP

How well did you know this?
1
Not at all
2
3
4
5
Perfectly
82
Q

ECG finding in MVP

A

most commonly normal but may show biphasic or inverted T-waves in leads II, III, and aVF, and occasionally supraventricular or ventricular premature beats

How well did you know this?
1
Not at all
2
3
4
5
Perfectly
83
Q

The physical examination hallmark of PR is a high-pitched, decrescendo diastolic murmur also called ______ is heard along the left sternal border

A

(Graham Steell murmur)

How well did you know this?
1
Not at all
2
3
4
5
Perfectly
84
Q

most common form of cyanotic CHD, occurring in 0.5 per 1000 live births.

A

Tetralogy of Fallot (TOF)

How well did you know this?
1
Not at all
2
3
4
5
Perfectly
85
Q

The base of the left lung may be compressed by pericardial fluid, producing _____ sign, a patch of dullness and increased fremitus beneath the angle of the left scapula.

A

Ewart’s sign

How well did you know this?
1
Not at all
2
3
4
5
Perfectly
86
Q

Treatment for pericarditis

A

There is no specific tx for acute idiopathic pericarditis, but bed rest and anti-inflammatory tx with aspirin (2–4 g/d), with NSAIDs, such as ibuprofen (600–800 mg tid) or indomethacin (25–50 mg tid), and should be administered along with gastric protection (e.g., omeprazole 20 mg/d)

colchicine (0.5 mg qd [70 kg]), should be administered for 3 months. Colchicine enhances the response to NSAIDs and also aids in reducing the risk of recurrent pericarditis

However, since they increase the risk of subsequent recurrence, full-dose corticosteroids should be given for only 2–4 days and then tapered. Anticoagulants should be avoided because their use could cause bleeding into the pericardial cavity and tamponade.

How well did you know this?
1
Not at all
2
3
4
5
Perfectly
87
Q

What constitutes Beck’s triad?

A

hypotension
soft or absent heart sounds
jugular venous distention

How well did you know this?
1
Not at all
2
3
4
5
Perfectly
88
Q

What constitutes an important feature in the differentiation of acute pericarditis from AMI

A

The almost simultaneous development of fever and precordial pain, often 10–12 days after a presumed viral illness

in AMI chest pain precedes fever.

How well did you know this?
1
Not at all
2
3
4
5
Perfectly
89
Q

What in the JVP wave form is absent or diminished in cardiac tamponade, is the most prominent deflection in constrictive pericarditis

A

y descent

How well did you know this?
1
Not at all
2
3
4
5
Perfectly
90
Q

In acute pericarditis, the apical pulse is reduced and may retract in systole AKA

A

(Broadbent’s sign)

How well did you know this?
1
Not at all
2
3
4
5
Perfectly
91
Q

the most common type of primary cardiac tumor in adults

A

Myxoma

How well did you know this?
1
Not at all
2
3
4
5
Perfectly
92
Q

In absolute terms, the most common primary sites from which cardiac metastases originate are carcinoma of the______ and _____ , reflecting the high incidence of those cancers

A

breast and lung

How well did you know this?
1
Not at all
2
3
4
5
Perfectly
93
Q

major site of atherosclerotic disease

A

Epicardial coronary arteries

How well did you know this?
1
Not at all
2
3
4
5
Perfectly
94
Q

Example of anginal equivalents

A

Anginal “equivalents” include dyspnea, nausea, fatigue, and faintness and are more common in the elderly and in diabetic patients

How well did you know this?
1
Not at all
2
3
4
5
Perfectly
94
Q

When a stenosis reduces the diameter of an epicardial artery by __%, there is a limitation of the ability to increase flow to meet increased myocardial demand.

When the diameter is reduced by ~___%, blood flow at rest may be reduced

A

50%, 80%

How well did you know this?
1
Not at all
2
3
4
5
Perfectly
95
Q

Contraindications for Ranolazine?

A

hepatic impairment
QTc prolongation
drugs that inhibit the CYP3A metabolic system (e.g., ketoconazole, diltiazem, verapamil, macrolide antibiotics, HIV protease inhibitors, and large quantities of grapefruit juice

How well did you know this?
1
Not at all
2
3
4
5
Perfectly
96
Q

MOA of nicorandil

A

opens ATP-sensitive potassium channels in myocytes

How well did you know this?
1
Not at all
2
3
4
5
Perfectly
97
Q

most common clinical indication for PCI is _____

A

symptom-limiting angina pectoris

How well did you know this?
1
Not at all
2
3
4
5
Perfectly
98
Q

the most common etiology of secondary hypertension

A

Primary renal disease

How well did you know this?
1
Not at all
2
3
4
5
Perfectly
99
Q

An ankle-brachial index <___ is considered diagnostic of PAD and is associated with >50% stenosis in at least one major lower limb vessel.

A

0.90

ABIs >1.40 indicate noncompressible arteries secondary to vascular calcification

How well did you know this?
1
Not at all
2
3
4
5
Perfectly
100
Q

Cardiovascular disease risk doubles for every __ mmHg increase in systolic and __mmHg increase in diastolic pressure.

A

20 mmHg
10 mmHg

How well did you know this?
1
Not at all
2
3
4
5
Perfectly
101
Q

Recommended criteria for a diagnosis of hypertension, based on 24-h blood pressure monitoring, are average awake blood pressure ≥ _____ mmHg and asleep blood pressure ≥___ mmHg.

A

135/85
120/75

How well did you know this?
1
Not at all
2
3
4
5
Perfectly
102
Q

A ratio >___ in conjunction with a plasma aldosterone concentration >____ pmol/L (>20 ng/dL) reportedly has a sensitivity of 90% and a specificity of 91% for an aldosterone-producing adenoma

A

30:1
555

How well did you know this?
1
Not at all
2
3
4
5
Perfectly
103
Q

______ is the most common congenital cardiovascular cause of hypertension

A

Coarctation of the aorta

How well did you know this?
1
Not at all
2
3
4
5
Perfectly
104
Q

for hypertensive urgency The initial goal of therapy is to reduce mean arterial blood pressure by no more than % within minutes to 2 h or to a blood pressure in the range of mmHg

A

25%
160/100–110

How well did you know this?
1
Not at all
2
3
4
5
Perfectly
105
Q

The two most common autosomal dominant genetic mutations are ____ and ____ for inherited thrombophilias

A

factor V Leiden
prothrombin gene mutation

How well did you know this?
1
Not at all
2
3
4
5
Perfectly
106
Q

most common acquired cause of thrombophilia and is associated with venous or arterial thrombosis.

A

Antiphospholipid antibody syndrome

How well did you know this?
1
Not at all
2
3
4
5
Perfectly
107
Q

in PEThe most common gas exchange abnormalities are ____ and ____, which represents the inefficiency of O2 transfer across the lungs.

A

arterial hypoxemia and an increased alveolar-arterial O2 tension gradient

How well did you know this?
1
Not at all
2
3
4
5
Perfectly
108
Q

Hallmarks of massive PE

A

Dyspnea, syncope, hypotension, and cyanosis

How well did you know this?
1
Not at all
2
3
4
5
Perfectly
109
Q

most common symptom of DVT

A

“charley horse” in the lower calf that persists and intensifies over several days

How well did you know this?
1
Not at all
2
3
4
5
Perfectly
110
Q

Signs of PE on CXR

A

Well-established abnormalities include focal oligemia (Westermark’s sign), a peripheral wedged-shaped density usually located at the pleural base (Hampton’s hump), and an enlarged right descending pulmonary artery (Palla’s sign).

How well did you know this?
1
Not at all
2
3
4
5
Perfectly
111
Q

Finding on chest on chest CT indicates an increased likelihood of death within the next 30 days

A

RV enlargement

How well did you know this?
1
Not at all
2
3
4
5
Perfectly
112
Q

pressure required for compression stockings for acute DVT

A

30-40 mmhg

How well did you know this?
1
Not at all
2
3
4
5
Perfectly
112
Q

Define McConnell’s sign

A

The best-known indirect sign of PE on transthoracic echocardiography is McConnell’s sign: hypokinesis of the RV free wall with normal or hyperkinetic motion of the RV apex

PE= apical sparing

How well did you know this?
1
Not at all
2
3
4
5
Perfectly
113
Q

For life-threatening or intracranial hemorrhage due to heparin or LMWH, what can be administered?

A

protamine sulfate can be administered

How well did you know this?
1
Not at all
2
3
4
5
Perfectly
114
Q

Antidote for dabigatran

A

idarucizumab

How well did you know this?
1
Not at all
2
3
4
5
Perfectly
115
Q

The only Food and Drug Administration–approved indication for PE fibrinolysis is ____

A

massive PE

How well did you know this?
1
Not at all
2
3
4
5
Perfectly
116
Q

Approximately 90% of syphilitic aneurysms are located in ______

A

ascending aorta or aortic arch

How well did you know this?
1
Not at all
2
3
4
5
Perfectly
117
Q

What are the indications for operative repair for ascending aorta aneurysm?

A

Operative repair with placement of a prosthetic graft is indicated in patients with symptomatic ascending thoracic aortic aneurysms and for most asymptomatic aneurysms, including those associated with bicuspid aortic valves, when the aortic root or ascending aortic diameter is ≥5.5 cm, or when the growth rate is >0.5 cm per year.

In patients with Marfan’s syndrome, ascending thoracic aortic aneurysms of 4–5 cm should be considered for surgery.

How well did you know this?
1
Not at all
2
3
4
5
Perfectly
118
Q

What are the indications for operative repair for descending aorta aneurysm?

A

Operative repair is indicated for patients with degenerative descending thoracic aortic aneurysms when the diameter is >6 cm, and endovascular repair should be considered if feasible when the diameter is >5.5 cm.

How well did you know this?
1
Not at all
2
3
4
5
Perfectly
119
Q

Target BP and HR for px with acute dissection

A

<= 120 mmHg
<60 bpm

How well did you know this?
1
Not at all
2
3
4
5
Perfectly
120
Q

What is a phosphodiesterase inhibitor with vasodilator and antiplatelet properties, increases claudication distance by 40–60% and improves measures of quality of life

A

Cilostazol

How well did you know this?
1
Not at all
2
3
4
5
Perfectly
121
Q

most common peripheral artery aneurysms

A

popliteal artery

How well did you know this?
1
Not at all
2
3
4
5
Perfectly
122
Q

Compression of a large arteriovenous fistula may cause reflex slowing of the heart rate AKA as ____ sign

A

(Nicoladoni-Branham sign)

How well did you know this?
1
Not at all
2
3
4
5
Perfectly
123
Q

principal diagnostic test to evaluate patients with chronic venous disease

A

venous duplex ultrasonography.

How well did you know this?
1
Not at all
2
3
4
5
Perfectly
124
Q

Graduated compression stockings with pressures of ____ mmHg are suitable for most patients with simple varicose veins, although pressures of ____ mmHg may be required for patients with manifestations of venous insufficiency such as edema and ulcers.

A

20–30
30–40

How well did you know this?
1
Not at all
2
3
4
5
Perfectly
125
Q

The most common etiology of secondary lymphedema worldwide is _______

A

lymphatic filariasis

How well did you know this?
1
Not at all
2
3
4
5
Perfectly
126
Q

Thickening of the skin in lymphedema is detected by ______sign, which is the inability to tent the skin at the base of the toes

A

Stemmer’s

How well did you know this?
1
Not at all
2
3
4
5
Perfectly
127
Q

Stage of lymphedema wherein fluid subsides with limb elevation

A

Stage I

How well did you know this?
1
Not at all
2
3
4
5
Perfectly
128
Q

Stage of lymphedema wherein fluid does not subside with limb elevation

A

Stage II

How well did you know this?
1
Not at all
2
3
4
5
Perfectly
129
Q

How do you define resistant hypertension?

A

BP persistently >140/90 despite use of 3 or more
antihypertensives including a diuretic

How well did you know this?
1
Not at all
2
3
4
5
Perfectly
130
Q

parameters is indicative of worse outcomes in
patients with acute decompensated heart failure
BUN
Crea
troponin levels
SBP

A

BUN > 43 mg/dL
Crea > 2.75 mg/dl
troponin levels- elevated
SBP < 115 mmHg

How well did you know this?
1
Not at all
2
3
4
5
Perfectly
131
Q

In acute MI, which papillary muscle is most commonly involved?

A

Posteromedial papillary muscle of the left ventricle

How well did you know this?
1
Not at all
2
3
4
5
Perfectly
132
Q

Patients who have angina when under emotional stress are classified under the Canadian Cardiovascular Society (CCS) as functional class:

A

II

How well did you know this?
1
Not at all
2
3
4
5
Perfectly
133
Q

MOA of Nicorandil

A

Stimulates K adenosine triphosphate channels

How well did you know this?
1
Not at all
2
3
4
5
Perfectly
134
Q

How long should you stop smoking before surgery to reduce perioperative pulmonary complications

A

8 weeks before and 10 days after

How well did you know this?
1
Not at all
2
3
4
5
Perfectly
135
Q

Most common cause of mid systolic murmur in adults

A

AS

How well did you know this?
1
Not at all
2
3
4
5
Perfectly
136
Q

Right sided sounds increase with inspiration except for ____

A

pulmonic eejction sounds

How well did you know this?
1
Not at all
2
3
4
5
Perfectly
137
Q

Most impt component of acute management of ADHF with high BP

A

Vasodilator since likely no due to volume overload

if normal bp likely due to volume overload hence need to give diuretics

How well did you know this?
1
Not at all
2
3
4
5
Perfectly
138
Q

What is the principal determinant of the clinical course, manifestation and feasibility of repeair in px with VSD

A

status of the pulmonary vascular bed

How well did you know this?
1
Not at all
2
3
4
5
Perfectly
139
Q

Pathologic finding in arterioles of target organs in malignant hypertension

A

fibrinoid necrosis

How well did you know this?
1
Not at all
2
3
4
5
Perfectly
140
Q

Recommended criteria for diagnosis of hypertension in 24 hr ambulatory bp

A

Average awake BP >= 135/85
Asleep BP >=120/70

approximate a clinic bp of 140/90

How well did you know this?
1
Not at all
2
3
4
5
Perfectly
141
Q

Anti anginal drug that should be avoided in HOCM

A

Nitroglycerin

How well did you know this?
1
Not at all
2
3
4
5
Perfectly
142
Q

Most commonly implicated drugs in toxic cardiomyopathy

A

Chemotherapeutic drugs

How well did you know this?
1
Not at all
2
3
4
5
Perfectly
143
Q

Threshold transferrin saturation for hemochromatosis induced cardiomyopathy

A

> 60% for men
45-50% for women

How well did you know this?
1
Not at all
2
3
4
5
Perfectly
144
Q

The usual first symptom in restrictive cardiomyopathy

A

Subtle exercise intolerance

How well did you know this?
1
Not at all
2
3
4
5
Perfectly
145
Q

Leading cause of SCD in the young

A

HOCM

How well did you know this?
1
Not at all
2
3
4
5
Perfectly
146
Q

Tuberculous aneurysm usually affects which segment of the aorta

A

Thoracic

T-B, T-horacic

How well did you know this?
1
Not at all
2
3
4
5
Perfectly
147
Q

Drug indicated in AAA to reduce cardiovascular events related to atherosclerosis

A

Statins

90% of AAA > 4cm are related to atherosclerotic dse

How well did you know this?
1
Not at all
2
3
4
5
Perfectly
148
Q

How do you differentiate the different Debakey types of aoric dissection

A
How well did you know this?
1
Not at all
2
3
4
5
Perfectly
149
Q

How do you differentiate the different Stanford types of aoric dissection

A
How well did you know this?
1
Not at all
2
3
4
5
Perfectly
150
Q

Most frequently cited ecg abnormality in PE

A

Sinus tachy
S1Q3T3

How well did you know this?
1
Not at all
2
3
4
5
Perfectly
151
Q

Most common ecg abnormality in PE

A

t wave inversion in v1-v4

How well did you know this?
1
Not at all
2
3
4
5
Perfectly
152
Q

Most typicl symptom of PAOD

A

intermittent claudication

How well did you know this?
1
Not at all
2
3
4
5
Perfectly
153
Q

Characteristic murmur of VSD

A

Holosystolic murmur heard best over 3rd and 4th interspaces along the sternal border
increases with hand grip

How well did you know this?
1
Not at all
2
3
4
5
Perfectly
154
Q

Characteristic murmur of PDA

A

Small: silent
Large: continuous machinery like murmur heard best below the left clavicle

How well did you know this?
1
Not at all
2
3
4
5
Perfectly
155
Q

Imaging of choice for follow up repaired TOF

A

Cardiac MRI

How well did you know this?
1
Not at all
2
3
4
5
Perfectly
156
Q

Associated valvular abnormality in straight back syndrome

A

MVP

How well did you know this?
1
Not at all
2
3
4
5
Perfectly
157
Q

which cardiac imaging modality has the highest sensitivity and specificity in diagnosing coronary artery disease?

A

CT angio

How well did you know this?
1
Not at all
2
3
4
5
Perfectly
157
Q

NT pro BNP can be falsely elevated in which circumstances?

A

older age
renal impairment
women
right HF
use of ARNIs

How well did you know this?
1
Not at all
2
3
4
5
Perfectly
158
Q

What drug therapy is useful across all types of
the most common valvular heart diseases?

A

diuretics

How well did you know this?
1
Not at all
2
3
4
5
Perfectly
159
Q

Triad of Buerger’s dse

A

Raynauds, claudication, migratory superficial vein phlebitis

How well did you know this?
1
Not at all
2
3
4
5
Perfectly
160
Q

What BP should you start treating the following
acute ischemic stroke
For thrombolysis
Hemorrhagic stroke

A

acute ischemic stroke: SBP >220 or DBP >130
For thrombolysis: lower to < 185/110
Hemorrhagic stroke: if SBP > 180 or DBP >130

How well did you know this?
1
Not at all
2
3
4
5
Perfectly
161
Q

How long is the target time from Door In to provincial hospital to Door Out of provincial hospital (DIDO) (to a PCI capable hospital)

A

30 mins

How well did you know this?
1
Not at all
2
3
4
5
Perfectly
162
Q

At which diameter of thoracic aortic aneurysm should a marfan’s syndrome patient be considered for operative repair

A

4-5 cm

How well did you know this?
1
Not at all
2
3
4
5
Perfectly
163
Q

Among patients receiving aggressive fluid resuscitation for extensive fluid losses, a decrease in renal output with rise in serum creatinine may indicate which complication of managemen

A

renal vein compression

How well did you know this?
1
Not at all
2
3
4
5
Perfectly
164
Q

in a patient with PAD what the best test to objectively assess his functional limitations?

A

Treadmill stress test

How well did you know this?
1
Not at all
2
3
4
5
Perfectly
165
Q

Sodium intake limit in px with hypertension

A

1500 mg/day

How well did you know this?
1
Not at all
2
3
4
5
Perfectly
166
Q

BP target for px with DM and hypertension

A

< 130/80

How well did you know this?
1
Not at all
2
3
4
5
Perfectly
167
Q

What electrolyte abnormality is predictive of worse outcome in HF?

A

Hyponatremia

How well did you know this?
1
Not at all
2
3
4
5
Perfectly
168
Q

Characteristics of vulnerable plaque

A

Vulnerable Plaques may show:
1. Eccentric Stenosis with scalloped or overhanging edges
2. Narrow neck on coronary angiography
3. Lipid-rich core and THIN Fibrous cap

How well did you know this?
1
Not at all
2
3
4
5
Perfectly
169
Q

Mid systolic click is associated with what condition

A

MVP

A frequent finding is the mid- or late (nonejection) systolic click, which occurs 0.14 s or more after S1 and is thought to be generated by the sudden tensing of slack, elongated chordae tendineae or by the prolapsing mitral leaflet when it reaches its maximal excursion. Systolic clicks may be multiple and may be followed by a high-pitched, mid-late systolic crescendo–decrescendo murmur, which occasionally is “whooping” or “honking” and is heard best at the apex. Radiation of the murmur will depend on the involved leaflet.

How well did you know this?
1
Not at all
2
3
4
5
Perfectly
170
Q

What medications can cause increased amplitude of u waves?

A

An abnormal increase in U-wave amplitude is most commonly due to drugs (e.g., dofetilide, amiodarone, sotalol, quinidine) or to hypokalemia.

How well did you know this?
1
Not at all
2
3
4
5
Perfectly
171
Q

the most common cause of marked left axis deviation in adult

A

Left anterior fascicular block (QRS axis more negative than –45°) is probably the most common cause of marked left axis deviation in adult

How well did you know this?
1
Not at all
2
3
4
5
Perfectly
172
Q

which diseases do you see differential cyanosis?

A

Differential cyanosis refers to isolated cyanosis affecting the lower but not the upper extremities in a patient with a large patent ductus arteriosus (PDA) and secondary pulmonary hypertension with right-to-left to shunting at the great vessel level.

How well did you know this?
1
Not at all
2
3
4
5
Perfectly
173
Q

What are the contrainidcations to stress testing?

A
How well did you know this?
1
Not at all
2
3
4
5
Perfectly
174
Q

What features during stress testing indicate severe IHD and high risk of future events?

A
How well did you know this?
1
Not at all
2
3
4
5
Perfectly
175
Q

Contraindications of prasugrel

A

prior stroke or transient ischemic attack or at high risk for bleedin

How well did you know this?
1
Not at all
2
3
4
5
Perfectly
176
Q

Most common cause of aortic stenosis

A

degenerative calcification

How well did you know this?
1
Not at all
2
3
4
5
Perfectly
177
Q

Observational studies show increased risk of CVD beginning with systolic blood pressures (SBPs) >_____- mmHg

A

110–115

How well did you know this?
1
Not at all
2
3
4
5
Perfectly
178
Q

NYHA classification

A
How well did you know this?
1
Not at all
2
3
4
5
Perfectly
179
Q

When should you request for 2d echo in px with murmur?

A
How well did you know this?
1
Not at all
2
3
4
5
Perfectly
180
Q

Asymptomatic or mildly symptomatic patients with valvular heart disease that is anatomically severe should be evaluated periodically, every _____ months, by clinical and noninvasive examinations

A

6–12

How well did you know this?
1
Not at all
2
3
4
5
Perfectly
181
Q

Conditions that prolong phase 2 or 3 increase/decrease the QT interval.

A

increase

eg. (amiodarone, hypocalcemia)

How well did you know this?
1
Not at all
2
3
4
5
Perfectly
182
Q

most common arrhythmia

A

sinus tachycardia

How well did you know this?
1
Not at all
2
3
4
5
Perfectly
183
Q

Biventricular pacing usually produce LBBB or CRBBB?

A

CRBBB

How well did you know this?
1
Not at all
2
3
4
5
Perfectly
184
Q

most common cause of marked left axis deviation in adults

A

LAFB

How well did you know this?
1
Not at all
2
3
4
5
Perfectly
185
Q

Prevention of contrast induced AKI

A

For all patients, adequate intravascular volume expansion with intravenous 0.9% saline (1.0–1.5 mL/kg per hour) for 3–12 h before and continued 6–24 h after the procedure limits the risk of contrast-induced acute kidney injury by >50%.

Pretreatment with N-acetylcysteine (Mucomyst) has not reduced the risk of contrast-induced acute kidney injury consistently and, therefore, is no longer recommended routinely.

Diabetic patients treated with metformin should stop the drug 24 h prior to the procedure and not restart until 48 h after contrast administration to limit the associated risk of lactic acidosis.

Other strategies to decrease risk include the administration of sodium bicarbonate (3 mL/kg per hour) 1 h before and 6 h after the procedure (similar outcome to saline infusion); use of low- or iso-osmolar contrast agents; and limiting the volume of contrast to <50 mL per procedure.

How well did you know this?
1
Not at all
2
3
4
5
Perfectly
186
Q

Cardiac catheterization is performed after the patient has fasted for _ h and has received intravenous conscious sedation to remain awake but sedated during the procedure

A

6

How well did you know this?
1
Not at all
2
3
4
5
Perfectly
187
Q

Test to do to confirm dual blood supply to the hand

A

A normal modified Allen’s test or Barbeau test confirming dual blood supply to the hand from the radial and ulnar arteries is recommended prior to access at this site

How well did you know this?
1
Not at all
2
3
4
5
Perfectly
188
Q

Systolic pulsations over the liver signify what valvular abnormality

A

severe tricuspid regurgitation (TR)

How well did you know this?
1
Not at all
2
3
4
5
Perfectly
189
Q

Clubbing implies the presence of central right-to-left shunting, although it has also been described in patients with ________

A

endocarditis

How well did you know this?
1
Not at all
2
3
4
5
Perfectly
190
Q

What sign signifies posterior calf pain on active dorsiflexion of the foot against resistance?

A

A Homan’s sign (posterior calf pain on active dorsiflexion of the foot against resistance) is neither specific nor sensitive for deep venous thrombosis

How well did you know this?
1
Not at all
2
3
4
5
Perfectly
191
Q

Why is IJV preferred over EJV for measuring JVP?

A

The internal jugular vein is preferred because the external jugular vein is valved and not directly in line with the superior vena cava and right atrium

How well did you know this?
1
Not at all
2
3
4
5
Perfectly
192
Q

Abnormal JVP

A

A distance >4.5 cm at 30° elevation is considered abnormal

How well did you know this?
1
Not at all
2
3
4
5
Perfectly
193
Q

Normally, the venous pressure should fall by at least ___ mmHg with inspiration.

A

3

Kussmaul’s sign is defined by either a rise or a lack of fall of the JVP with inspiration and is classically associated with constrictive pericarditis, although it has been reported in patients with restrictive cardiomyopathy, massive pulmonary embolism, right ventricular infarction, and advanced left ventricular (LV) systolic heart failure

How well did you know this?
1
Not at all
2
3
4
5
Perfectly
194
Q

Abdominojugular reflux is produced with firm and consistent pressure over the upper portion of the abdomen, preferably over the right upper quadrant, for >__s.

A positive response is defined by a sustained rise of >3 cm in the JVP during the application of firm abdominal pressure

A

15

The response should be assessed after 10 s of continuous pressure to allow for respiratory artifacts and tensing of the abdominal muscles to subside.

Kussmaul’s sign is defined by either a rise or a lack of fall of the JVP with inspiration and is classically associated with constrictive pericarditis, although it has been reported in patients with restrictive cardiomyopathy, massive pulmonary embolism, right ventricular infarction, and advanced left ventricular (LV) systolic heart failure.

It is also a common, isolated finding in patients after cardiac surgery without other hemodynamic abnormalities.

How well did you know this?
1
Not at all
2
3
4
5
Perfectly
195
Q

When taking the BP, The cuff should be inflated to __ mmHg above the expected systolic pressure and the pressure released at a rate of ___ mmHg/s

A

30

2-3

How well did you know this?
1
Not at all
2
3
4
5
Perfectly
196
Q

Very low (even 0 mmHg) diastolic blood pressures may be recorded in patients with ___________ because of enhanced diastolic “run-off.”

A

chronic, severe AR or a large arteriovenous fistula

How well did you know this?
1
Not at all
2
3
4
5
Perfectly
197
Q

Blood pressure should be measured in both arms, and the difference should be <___ mmHg

A

10

Systolic leg pressures are usually as much as 20 mmHg higher than systolic arm pressures. Greater leg–arm pressure differences are seen in patients with chronic severe AR as well as patients with extensive and calcified lower extremity peripheral arterial disease

How well did you know this?
1
Not at all
2
3
4
5
Perfectly
198
Q

Definition of white coat hypertension

A

“White coat hypertension” (elevated clinic blood pressure and normal out of clinic blood pressure) is defined by at least three separate clinic-based measurements >130/80 mmHg and at least two non-clinic-based measurements <130/80 mmHg in the absence of any evidence of target organ damagef

How well did you know this?
1
Not at all
2
3
4
5
Perfectly
199
Q

Definition of orthostatic hypotensio

A

Orthostatic hypotension is defined by a fall in systolic pressure >20 mmHg or in diastolic pressure >10 mmHg in response to assumption of the upright posture from a supine position within 3 min

How well did you know this?
1
Not at all
2
3
4
5
Perfectly
200
Q

The aortic pulse is best appreciated in the _______

A

The aortic pulse is best appreciated in the epigastrium, just above the level of the umbilicus

How well did you know this?
1
Not at all
2
3
4
5
Perfectly
201
Q

A bifid pulse is also described in patients with _______, with inscription of percussion and tidal waves

A

hypertrophic obstructive cardiomyopathy (HOCM)

A bifid pulse is easily appreciated in patients on intraaortic balloon counterpulsation (IABP), in whom the second pulse is diastolic in timing

Also seen in advanced AR

How well did you know this?
1
Not at all
2
3
4
5
Perfectly
202
Q

Definition of pulsus paradoxus

A

Pulsus paradoxus refers to a fall in systolic pressure >10 mmHg with inspiration that is seen in patients with pericardial tamponade but also is described in those with massive pulmonary embolism, hemorrhagic shock, severe obstructive lung disease, and tension pneumothorax.

How well did you know this?
1
Not at all
2
3
4
5
Perfectly
203
Q

Sustained apex beat is present in which conditions

A

A sustained apex beat is a sign of pressure overload, such as that which may be present in patients with AS or chronic hypertension

How well did you know this?
1
Not at all
2
3
4
5
Perfectly
204
Q

An unusually narrowly split or even a singular S2 is a feature of ______

A

pulmonary arterial hypertension

How well did you know this?
1
Not at all
2
3
4
5
Perfectly
205
Q

Fixed splitting of S2 , in which the A2 –P2 interval is wide and does not change during the respiratory cycle, occurs in patients with a

A

secundum atrial septal defect

How well did you know this?
1
Not at all
2
3
4
5
Perfectly
206
Q

The pericardial knock (PK) is also high-pitched and occurs slightly later than the OS, corresponding in

A

timing to the abrupt cessation of ventricular expansion after tricuspid valve opening and to an exaggerated y descent seen in the jugular venous waveform in patients with constrictive pericarditis

How well did you know this?
1
Not at all
2
3
4
5
Perfectly
207
Q

two types of benign continuous murmurs.

A

The cervical venous hum is heard in children or adolescents in the supraclavicular fossa. It can be obliterated with firm pressure applied to the diaphragm of the stethoscope, especially when the subject turns his or her head toward the examiner.

The mammary soufflé of pregnancy relates to enhanced arterial blood flow through engorged breasts

No further work ups needed

How well did you know this?
1
Not at all
2
3
4
5
Perfectly
208
Q

Right-sided murmurs and sounds generally increase with inspiration, except for

A

Pulmonary ejection sounds

How well did you know this?
1
Not at all
2
3
4
5
Perfectly
209
Q

A change in the intensity of a systolic murmur in the first beat after a premature beat or in the beat after a long cycle length in patients with atrial fibrillation suggests valvular AS rather than MR, particularly in an older patient in whom the murmur of the AS may be well transmitted to the apex AKA ______ effect

A

(Gallavardin effect)

How well did you know this?
1
Not at all
2
3
4
5
Perfectly
210
Q

a potent vasoconstrictor peptide with growth-promoting effects that may play an important role in pulmonary hypertension and right ventricular failure

A

Endothelin

How well did you know this?
1
Not at all
2
3
4
5
Perfectly
211
Q

chronic anemia is associated with high CO when hemoglobin reduces significantly, for example, to a level that is ≤

A

8 g/dL.

How well did you know this?
1
Not at all
2
3
4
5
Perfectly
212
Q

Definition of cardiac cachexia

A
How well did you know this?
1
Not at all
2
3
4
5
Perfectly
213
Q

In patients with chronic HF on guideline-directed medical therapy, resting heart rate ideally should be <___ beats/min, and blood pressure should be in the normal to low-normal range

A

70–75

How well did you know this?
1
Not at all
2
3
4
5
Perfectly
214
Q

T/F You should screen px with HF for anxiety

A

False

Depression

Depression is an independent risk factor for adverse outcomes in HF , especially in older women.

The AHA recommends screening for depression among patients with cardiovascular disease including HF using validated patient health questionnaires. Selective serotonin reuptake inhibitors are safe for treating depression in HF but do not appear to affect the natural history of disease

How well did you know this?
1
Not at all
2
3
4
5
Perfectly
215
Q

What is the CHARM trial about?

A

The Candesartan in Heart Failure—Assessment of Mortality and Morbidity (CHARM) Preserved study showed a statistically significant reduction in HF hospitalizations but no difference in all-cause mortality in patients with HFpEF who were treated with the ARB candesartan

How well did you know this?
1
Not at all
2
3
4
5
Perfectly
216
Q

What is the I-PRESERVE trial about?

A

Irbesartan in Heart Failure with Preserved Systolic Function (I-PRESERVE) trial demonstrated no differences in the composite of cardiovascular death or HF hospitalization during treatment with the ARB irbesartan compared with placebo.

How well did you know this?
1
Not at all
2
3
4
5
Perfectly
217
Q

What is the PEP-CHF trial about?

A

Apparent early benefits of the ACE inhibitor perindopril on HF hospitalizations and functional capacity in the Perindopril in Elderly People with Chronic Heart Failure (PEP-CHF) study were attenuated over longer-duration follow-up

How well did you know this?
1
Not at all
2
3
4
5
Perfectly
218
Q

What is the DIG trial about?

A

The Digitalis Investigation Group (DIG) Ancillary Trial found no impact of digoxin on all-cause mortality or on all-cause or cardiovascular hospitalization among patients with chronic HF, ejection fraction (EF) >45%, and sinus rhythm, although a modest reduction in HF hospitalizations was noted.

How well did you know this?
1
Not at all
2
3
4
5
Perfectly
219
Q

What is the SENIORS trial about?

A

the subgroup of elderly patients with prior hospitalization and HFpEF enrolled in the Study of the Effects of Nebivolol Intervention on Outcomes and Rehospitalization in Seniors with Heart Failure (SENIORS) trial of nebivolol, a vasodilating beta blocker, did not appear to experience significant reductions in all-cause or cardiovascular mortality.

How well did you know this?
1
Not at all
2
3
4
5
Perfectly
220
Q

What is the TOPCAT trial about?

A

Treatment of Preserved Cardiac Function Heart Failure with an Aldosterone Antagonist (TOPCAT) trial explored the potential benefit of spironolactone compared to placebo in HFpEF. This trial demonstrated no improvement in the primary composite endpoint of cardiovascular death, HF hospitalizations, or aborted cardiac arrest but did show a reduction in HF hospitalizations among those allocated to spironolactone

How well did you know this?
1
Not at all
2
3
4
5
Perfectly
221
Q

What is the RELAX trial about?

A

This finding led to the phase 2 trial, Phosphodiesterase-5 Inhibition to Improve Clinical Status and Exercise Capacity in Diastolic Heart Failure (RELAX), in HFpEF patients (left ventricular EF [LVEF] >50%) with New York Heart Association (NYHA) functional class II or III symptoms, who received sildenafil at 20 mg three times daily for 3 months, followed by 60 mg three times daily for another 3 months, compared with a placebo. There was no improvement in functional capacity, QOL, or other clinical and surrogate parameters in those allocated to sildenafil compared to placebo.

How well did you know this?
1
Not at all
2
3
4
5
Perfectly
222
Q

What is the NEAT-HFpEF trial about?

A

On the premise that nitrates, which are NO donors, might improve preload, coronary perfusion, endothelial function, and exercise tolerance, the Nitrate’s Effect on Activity Tolerance in Heart Failure with Preserved Ejection Fraction (NEAT-HFpEF) study was conducted. Isosorbide mononitrate did not improve QOL or submaximal exercise capacity and decreased overall activity levels in treated patients

How well did you know this?
1
Not at all
2
3
4
5
Perfectly
223
Q

What is the INDIE-HFpEF trial about?

A

Inorganic nitrate compounds have also been shown to enhance NO signaling but did not improve functional capacity compared to placebo among patients with HFpEF randomized in the Inorganic Nitrite Delivery to Improve Exercise Capacity in Heart Failure with Preserved Ejection Fraction (INDIE-HFpEF) trial.

How well did you know this?
1
Not at all
2
3
4
5
Perfectly
224
Q

What is the PARAGON-HF trial about?

A

The PARAGON-HF trial randomized 4822 patients with symptomatic HFpEF (LVEF ≥45%), elevated natriuretic peptides, and structural heart disease to treatment with either sacubitril-valsartan or valsartan with the novel composite primary endpoint of cardiovascular death and total hospitalizations for HF. Although there was a 13% reduction in the rate of the primary composite endpoint in those allocated to sacubitril-valsartan, this result narrowly missed the margin for statistical significance in the primary statistical analysis (p = .06).

Directional benefits in secondary endpoints including QOL, NYHA class, and renal function favoring sacubitril-valsartan support a possible modest benefit of neprilysin inhibition in this population, particularly among patients with lower (i.e., mildly reduced or mid-range) EF and women, subgroups who appeared to derive greater benefit.

How well did you know this?
1
Not at all
2
3
4
5
Perfectly
225
Q

What is the DAPA-HF trial about?

A

Addition of the SGLT-2 inhibitor dapagliflozin to guideline-directed medical therapy of HFrEF was associated with reductions in cardiovascular mortality and HF hospitalization among patients with and without diabetes enrolled in the Dapagliflozin and Prevention of Adverse Outcomes in Heart Failure (DAPA-HF) study

How well did you know this?
1
Not at all
2
3
4
5
Perfectly
226
Q

T/F
Predischarge measurement of natriuretic peptide levels, which are highly correlated with risk for postdischarge mortality and readmission, may also be useful in assessing the adequacy of therapy and stratifying risk.

A

True

How well did you know this?
1
Not at all
2
3
4
5
Perfectly
227
Q

What is the CARRESS-HF trial about?

A

In the Cardiorenal Rescue Study in Acute Decompensated Heart Failure (CARRESS-HF) trial, 188 patients with ADHF and worsening renal failure were randomized to stepped pharmacologic care or UF. The primary endpoint was a change in serum creatinine and change in weight (reflecting fluid removal) at 96 h. Although similar weight loss occurred in both groups (~5.5 kg), there was a rise in serum creatinine among patients allocated to the UF group. Deaths and hospitalizations for HF were no different between groups, but there were more adverse events in the UF group, mainly due to kidney failure, bleeding complications, and intravenous catheter-related complications. This investigation argues against using UF as a primary strategy in patients with ADHF who are diuretic-responsive.

How well did you know this?
1
Not at all
2
3
4
5
Perfectly
228
Q

What is the ASCEND-HF trial about?

A

Clinical utilization of nesiritide has waned due to concerns raised regarding heightened risks of renal insufficiency and mortality identified in early trials. The Acute Study of Clinical Effectiveness of Nesiritide in Decompensated Heart Failure (ASCEND-HF) study randomizing 7141 patients with ADHF to nesiritide or placebo did not confirm this risk, but also identified no clear clinical benefit with regard to subsequent HF admissions, mortality, or symptom relief (reduction in dyspnea)

A smaller study of low-dose nesiritide in acute HF (Renal Optimization Strategies Evaluation Acute Heart Failure Study [ROSE-AHF]) also showed no incremental benefit over intravenous diuretics for relief of congestion or preservation of renal function.

How well did you know this?
1
Not at all
2
3
4
5
Perfectly
229
Q

What is the RELAX-AHF trial about?

A

In the Relaxin in Acute Heart Failure (RELAX-AHF) trial, 1161 patients hospitalized with ADHF, evidence of congestion, and systolic pressure >125 mmHg were randomized to treatment with serelaxin or placebo in addition to standard HF therapy. Serelaxin improved dyspnea, reduced signs and symptoms of congestion, and was associated with less early worsening of HF. A positive signal of reduced mortality identified in an exploratory analysis prompted a second study (RELAX-AHF2), which did not confirm an effect on cardiovascular death or worsening HF. Accordingly, this agent was not approved for use in clinical practice

How well did you know this?
1
Not at all
2
3
4
5
Perfectly
230
Q

What is the TRUE-AHF trial about?

A

the Trial of Ularitide Safety and Efficacy in Acute Heart Failure (TRUE-AHF) randomly allocated 2157 patients with acute HF to early treatment with the synthetic natriuretic peptide ularitide (at a dose sufficient to reduce ventricular wall stress) or placebo. Despite a very short duration between initial clinical presentation and pharmacologic intervention (<6 h) and early hemodynamic benefits, no improvement in clinical outcomes was observed in patients allocated to ularitide at 6 months. Ularitide was associated with a higher rate of hypotension and worsening serum creatinine

How well did you know this?
1
Not at all
2
3
4
5
Perfectly
231
Q

Dose of dopamine which exhibit
a. vasodilatory effects
b. beta adrenergic effects
c. alpha adrenergic effects

A

Dopamine exhibits dose-dependent effects on dopaminergic, α-, and β-adrenergic receptors, with vasodilatory effects predominating at lower doses (<2 μg/kg per min), β-adrenergic (inotropic) effects at moderate doses, and α-adrenergic effects (vasoconstriction) at higher doses (typically >10 μg/kg per min)

How well did you know this?
1
Not at all
2
3
4
5
Perfectly
232
Q

What is the OPTIME-CHF trial about?

A

Routine, short-term use of milrinone in patients hospitalized with ADHF in the Outcomes of a Prospective Trial of Intravenous Milrinone for Exacerbations of Chronic Heart Failure (OPTIME-CHF) trial was associated with increased risk of atrial arrhythmias and prolonged hypotension, but no benefit with regard to subsequent mortality or HF hospitalization. Accordingly, routine use of inotropic support in ADHF is discouraged, and these agents are currently indicated principally for short-term use as bridge therapy (to either left ventricular assist device support or to transplant) in cardiogenic shock or as selectively applied palliation in end-stage HF

How well did you know this?
1
Not at all
2
3
4
5
Perfectly
233
Q

a calcium sensitizer that provides inotropic activity but also possesses phosphodiesterase-3 inhibition properties that are vasodilatory

A

Levosimendan

Two trials, the second Randomized Multicenter Evaluation of Intravenous Levosimendan Efficacy (REVIVE II) and Survival of Patients with Acute Heart Failure in Need of Intravenous Inotropic Support (SURVIVE), have tested this agent in ADHF. SURVIVE compared levosimendan with dobutamine, and despite an initial reduction in circulating B-type natriuretic peptide levels in the levosimendan group compared with patients in the dobutamine group, this drug did not reduce all-cause mortality at 180 days or affect any secondary clinical outcomes.

The second trial compared levosimendan against traditional noninotropic therapy and found a modest improvement in symptoms with worsened short-term mortality and ventricular arrhythmias. Although levosimendan has been approved for use to support management of HF in several countries worldwide, it is not approved for use in the United States, largely owing to the lack of compelling data for incremental efficacy in comparison with conventional inotropic drugs or standard HF therapies

How well did you know this?
1
Not at all
2
3
4
5
Perfectly
234
Q

What is the PROTECT trial about?

A

Among patients with acute HF and renal dysfunction enrolled in the Placebo-Controlled Randomized Study of the Selective A1 Adenosine Receptor Antagonist Rolofylline for Patients Hospitalized with Acute Decompensated Heart Failure and Volume Overload to Assess Treatment Effect on Congestion and Renal Function (c) trial, no cardiovascular or renal benefit was observed.

How well did you know this?
1
Not at all
2
3
4
5
Perfectly
235
Q

What is the EVEREST trial about?

A

Similarly, despite compelling theoretical benefit of vasopressin receptor antagonism in acute HF (based on the central role of vasopressin in mediating the fluid retention that contributes to worsening HF), no benefit of the oral selective vasopressin-2 antagonist tolvaptan was seen with regard to mortality or HF-associated morbidity in the Efficacy of Vasopressin Antagonism in Heart Failure Outcome Study with Tolvaptan (EVEREST) trial.

How well did you know this?
1
Not at all
2
3
4
5
Perfectly
236
Q

What is the PIONEER trial about?

A

Comparison of Sacubitril-Valsartan Versus Enalapril on Effect on NT-proBNP in Patients Stabilized from an Acute Heart Failure Episode (PIONEER-HF) study of patients with HFrEF stabilized after hospital admission for ADHF, predischarge initiation of sacubitril-valsartan compared with enalapril was associated with greater reductions in natriuretic peptides as well as lower rates of composite death and HF readmission at 8 weeks.

How well did you know this?
1
Not at all
2
3
4
5
Perfectly
237
Q

What vasodilator is associated with Thiocyanate toxicity in renal insufficiency (>72 h)

A

Nitroprusside

** Also Requires arterial line placement for titration for precise blood pressure
management and prevention of hypotension

How well did you know this?
1
Not at all
2
3
4
5
Perfectly
238
Q

What is the CIBIS trial about?

A

Whether beta blockers or ACEIs should be started first was answered by the Cardiac Insufficiency Bisoprolol Study (CIBIS) III, in which outcomes did not vary based on the sequence of drug initiation. Thus, it matters little which agent is initiated first; what does matter is that optimally titrated doses of both ACEIs and beta blockers be established in a timely manner.

How well did you know this?
1
Not at all
2
3
4
5
Perfectly
239
Q

What trials were the basis for aggressive titration of HF medications to target doses?

A

Prospective trials of high- versus low-dose ACEIs (ATLAS), ARBs (HEAAL), and beta blockers (MOCHA) consistently favor the higher dose, with lower rates of death and HF hospitalization seen in the higher-dose group. Clinical experience suggests that, in the absence of symptoms to suggest hypotension (fatigue and dizziness), pharmacotherapy may be uptitrated every 2 weeks in stable ambulatory patients as tolerated

How well did you know this?
1
Not at all
2
3
4
5
Perfectly
240
Q

What is the RALES trial about

A

Spironolactone is the most commonly utilized agent in this class based on efficacy demonstrated in the Randomized Aldactone Evaluation Study (RALES) in patients with HFrEF and NYHA class III–IV symptoms. Eplerenone (studied principally in patients with milder NYHA class II symptoms and those with HF or left ventricular dysfunction complication myocardial infarction) lacks the antiandrogen effects of spironolactone and may be a suitable alternative for patients who experience sexual side effects (gynecomastia, erectile dysfunction, diminished libido)

How well did you know this?
1
Not at all
2
3
4
5
Perfectly
241
Q

Trials for HF about dual RAAS blockade

A

In both the Valsartan Heart Failure Trial (Val-HeFT) and the Candesartan in Heart Failure Assessment of Reduction in Mortality and Morbidity (CHARM-Added) trial, addition of an ARB to an ACEI and other HF therapy was associated with a lower risk of HF hospitalizations

Subsequent data from the Valsartan in Acute Myocardial Infarction (VALIANT) trial suggested that the addition of the ARB valsartan to an evidence-based dose of the ACEI captopril in patients with HF complicating myocardial infarction was associated with an increase in adverse events without any added benefit compared with monotherapy for either group.

The findings of the VALIANT trial are buttressed by more recent data from the Aliskiren Trial to Minimize Outcomes in Patients with Heart Failure (ATMOSPHERE), which randomly allocated 7016 patients with HFrEF to treatment with enalapril (targeted dose 10 mg twice daily as recommended by guidelines), the plasma renin inhibitor aliskiren, or the combination on top of standard HF therapy. In that study, combination treatment with aliskiren and enalapril was associated with higher rates of hyperkalemia, hypotension, and worsening renal function, but no incremental benefit with regard to HF hospitalization or cardiovascular mortality

How well did you know this?
1
Not at all
2
3
4
5
Perfectly
242
Q

What is the A-HeFT trial about?

A

A trial conducted in self-identified African Americans, the African-American Heart Failure Trial (A-HeFT), studied a fixed dose of isosorbide dinitrate with hydralazine in patients with advanced symptoms of HFrEF who were receiving standard background therapy including an ACEI and beta blocker. The study demonstrated improvements in survival and hospital admission for HF in the treatment group. Adherence to this regimen is limited by the thrice-daily dosing schedule

How well did you know this?
1
Not at all
2
3
4
5
Perfectly
243
Q

What is the OVERTURE trial about?

A

The combined drug omapatrilat hybridizes an ACEI with a neutral endopeptidase (neprilysin) inhibitor, and this agent was tested in the Omapatrilat Versus Enalapril Randomized Trial of Utility in Reducing Events (OVERTURE) trial. This drug did not favorably influence the primary outcome measure of the combined risk of death or hospitalization for HF requiring intravenous treatment compared with enalapril alone, and notably, the risk of angioedema was increased in patients assigned to omapatrilat

How well did you know this?
1
Not at all
2
3
4
5
Perfectly
244
Q

What is the effect of bosentan on HFrEF px?

A

. As an example, the endothelin antagonist bosentan is associated with worsening HF in HFrEF despite demonstrating benefits in right-sided HF due to pulmonary arterial hypertension.

Similarly, the centrally acting sympatholytic agent moxonidine worsens outcomes in left HF.

245
Q

What is the PARADIGM-HF trial about?

A

In the PARADIGM-HF trial, 8399 patients with HFrEF treated with guideline-directed medical therapy were randomly allocated to treatment with either enalapril or sacubitril-valsartan after a run-in period designed to ensure tolerability of both drugs at target doses. Compared to those assigned to enalapril, patients assigned to sacubitril-valsartan experienced a dramatic 20% reduction in the composite primary endpoint of cardiovascular death or HF hospitalization and a 16% reduction in all-cause mortality, as well as clinically important improvements in QOL measures. Sacubitril valsartan was well tolerated and associated with lower rates of hyperkalemia and worsening renal function, but greater rates of symptomatic hypotension, than enalapril. Guidelines now advocate a switch to ARNI for patients with symptomatic HFrEF who tolerate ACEIs and ARBs, and emerging data suggest that up-front utilization of ARNI in patients with de novo HF naïve to ACEIs/ARBs may also be appropriate for those with adequate blood pressure to tolerate it

246
Q

What is the SHIFT trial about?

A

The Systolic Heart Failure Treatment with Ivabradine Compared with Placebo Trial (SHIFT) was conducted in patients with NYHA class II or III HFrEF, prior HF hospitalization, sinus rhythm, and heart rate >70 beats/min. Ivabradine reduced the combined endpoint of cardiovascular-related death and HF hospitalization in proportion to the degree of heart rate reduction, which supports the notion that heart rate may be a therapeutic target in patients with HFrEF in sinus rhythm

247
Q

What is the EMPAREG trial about

A

A particular signal of benefit has been seen with regard to the incidence of HF hospitalization, which was reduced by 35% in comparison to placebo in the Empagliflozin Cardiovascular Outcome Event Trial in Type 2 Diabetes Mellitus Patients (EMPA-REG OUTCOMES) study

These results have been reinforced by the results of the EMPEROR-Reduced trial, in which 3730 patients with symptomatic HF and ejection fraction of 40% or less were randomized to treatment with empagliflozin (dosage 10 mg once daily) or placebo in addition to recommended therapy. Over median 16 month follow up, patients allocated to empagliflozin experienced a 25% reduction in the primary composite endpoint of cardiovascular death or hospitalization for HF, an effect that was again consistent regardless of the presence or absence of diabetes mellitus.

248
Q

primary presentation for fulminant myocarditis

A

Acute cardiogenic shock is the primary presentation for fulminant myocarditis, which can occur in otherwise healthy young adults and require rapid diagnosis and aggressive support, after which cardiac function may improve to near-normal levels.

249
Q

Most familial cardiomyopathies are inherited in an ______ pattern

A

autosomal dominant pattern

250
Q

initial evaluation for suspected myocarditis includes

A

The initial evaluation for suspected myocarditis includes an ECG, an echocardiogram, and serum levels of troponin and creatine phosphokinase, of which both cardiac and skeletal muscle fractions may be elevated. Magnetic resonance imaging is increasingly used for the diagnosis of myocarditis, which is supported but not proven by evidence of increased tissue edema and gadolinium enhancement , particularly in the mid-wall (as distinct from usual coronary artery territories)

251
Q

When biopsy is performed, the key Dallas criteria for myocarditis include

A

lymphocytic infiltrate with evidence of myocyte necrosis and are negative in 80–90% of patients with clinical myocarditis

Negative Dallas criteria can reflect sampling error or early resolution of lymphocytic infiltrates, but may also be influenced by the insensitivity of the test when inflammation results from cytokines and antibody-mediated injury. Routine histologic examination of endomyocardial biopsy rarely reveals a specific infective etiology, such as toxoplasmosis or cytomegalovirus subsets

252
Q

T/F The treatment of choice for Chagas’ cardiomyopathy is benznidazole

A

Moreover, in a large trial of adults with established Chagas’ cardiomyopathy, benznidazole did not prevent disease progression, leaving the role of antiparasitic therapy unclear.

253
Q

Treatment for cardiomyopathy caused by Trichinella

A

Treatment includes antihelminthic drugs (albendazole, mebendazole) and glucocorticoids if inflammation is severe

254
Q

What is the VICTORIA trial about?

A

The Vericiguat Global Study in Subjects with Heart Failure with Reduced Ejection Fraction (VICTORIA) randomly assigned 5050 patients with chronic HF, NYHA class II–IV symptoms, LVEF <45%, elevated natriuretic peptide levels, and evidence of worsening HF (requiring recent hospitalization or intravenous diuretic therapy) despite guideline-directed medical therapy to treatment with vericiguat (target dose 10 mg) or matching placebo over a median follow-up of 11 months. The primary study results were notable for a modest 10% relative risk reduction in the primary composite outcome of cardiovascular death or HF hospitalization among thos assigned to vericiguat, an effect driven principally by effects on HF hospitalization, rather than cardiovascular death

255
Q

What trials demonstrated improvent in HF outcomes using omecamtiv?

A

In the COSMIC-HF (Chronic Oral Study of Myosin Activation to Increase Contractility in Heart Failure) trial of 448 patients with chronic HF and left ventricular systolic dysfunction, treatment with omecamtiv mecarbil for 20 weeks was associated with significant improvements in cardiac function and indices of left ventricular remodeling, as well as reductions in natriuretic peptide levels. Notably, the safety profile was comparable to placebo, with no increase in cardiac adverse events despite a modest increase in cardiac troponins in patients allocated to omecamtiv mecarbil.

These promising preliminary data fueled a larger clinical outcomes trial (GALACTIC-HF, in which 8256 patients with symptomatic chronic heart failure and ejection fraction of 35% or less were randomized to treatment with omecamtiv mecarbil (dosage 25-50 mg twice daily based on plasma levels) or placebo in addition to standard HF therapy. Over median follow up of 21.8 months, patients allocated to omecamtiv mecarbil experienced a 14% reduction in the primary composite endpoint of death from cardiovascular causes or first heart failure event (hospitalization or urgent visit for heart failure), an outcome driven principally by reduction in heart failure events (no measureable effect on CV death alone)

256
Q

Trials about Rosuvastatin and HFrEF

A

Two trials, Controlled Rosuvastatin Multinational Trial in Heart Failure (CORONA) and Gruppo Italiano per lo Studio della Sopravvivenza nell’Insufficienza Cardiac (GISSI-HF), have tested low-dose rosuvastatin in patients with HFrEF and demonstrated no improvement in aggregate clinical outcomes

257
Q

What is the WARCEF trial about?

A

In the large Warfarin versus Aspirin in Reduced Cardiac Ejection Fraction (WARCEF) trial, full-dose aspirin or international normalized ratio–controlled warfarin was tested with follow-up for 6 years. Among patients with reduced LVEF in sinus rhythm, there was no significant overall difference in the primary outcome between treatment with warfarin and treatment with aspirin

258
Q

What is the PEECH trial about?

A

The Prospective Evaluation of Enhanced External Counterpulsation in Congestive Heart Failure (PEECH) study assessed the benefits of enhanced external counterpulsation in the treatment of patients with mild-to-moderate HF. This randomized trial improved exercise tolerance, QOL, and NYHA functional classification but without an accompanying increase in peak oxygen consumption. A placebo effect due to the nature of the intervention simply cannot be excluded

259
Q

What is the HF ACTION trial about?

A

The Heart Failure: A Controlled Trial Investigating Outcomes of Exercise Training (HF-ACTION) study investigated short-term (3-month) and long-term (12-month) effects of a supervised exercise training program in patients with moderate HFrEF. Exercise was safe, improved patients’ sense of well-being, and correlated with a trend toward mortality reduction. Maximal changes in 6-min walk distance were evident at 3 months with significant improvements in cardiopulmonary exercise time and peak oxygen consumption persisting at 12 months. Therefore, exercise training is recommended as an adjunctive treatment in patients with HF

260
Q

Trials about the benifits of ferric carboxymaltose and HF

A

Intravenous iron using either iron sucrose or carboxymaltose (Ferric Carboxymaltose Assessment in Patients with Iron Deficiency and Chronic Heart Failure [FAIR-HF] trial) has been shown to correct anemia and improve functional capacity.

Another trial, CONFIRM-HF, enrolled similar patients with iron deficiency (ferritin <100 ng/mL or 100–300 ng/mL if transferrin saturation <20%) and demonstrated that use of ferric carboxymaltose in a simplified high-dose schedule resulted in improvement in functional capacity, symptoms, and QOL.

261
Q

What is the SADHART trial about?

A

the largest randomized study of depression in HFrEF, the Sertraline Against Depression and Heart Disease in Chronic Heart Failure (SADHART-CHF) trial, showed that although sertraline was safe, it did not provide greater reduction in depression or improve cardiovascular status among patients with HF and depression compared with nurse-driven multidisciplinary management

262
Q

What is the ANDROMEDA trial about?

A

The Antiarrhythmic Trial with Dronedarone in Moderate-to-Severe Congestive Heart Failure Evaluating Morbidity Decrease (ANDROMEDA) studied the effects of the novel antiarrhythmic agent dronedarone and found an increased mortality due to worsening HF

263
Q

Trials that support CRT for HF

A

The Cardiac Resynchronization in Heart Failure Study (CARE-HF) trial was the first study to demonstrate a reduction in allcause mortality with CRT placement in patients with HFrEF on optimal therapy with continued moderate-to-severe residual symptoms of NYHA class III or IV HF.

More recent clinical trials have demonstrated disease-modifying properties of CRT in even minimally symptomatic patients with HFrEF, including the Resynchronization–Defibrillation for Ambulatory Heart Failure Trial (RAFT) and Multicenter Automatic Defibrillator Implantation Trial with Cardiac Resynchronization Therapy (MADIT-CRT), both of which sought to use CRT in combination with an implantable defibrillator. Most benefit in mildly symptomatic HFrEF patients accrues from applying this therapy in those with a QRS width of >149 ms and a left bundle branch block pattern

264
Q

What is the STITCH trial about?

A

The Surgical Treatment for Ischemic Heart Failure (STICH) trial in patients with an EF of ≤35% and coronary artery disease amenable to CABG demonstrated no significant initial benefit compared to medical therapy. However, patients assigned to CABG had lower rates of death from cardiovascular causes and of death from any cause or hospitalization for cardiovascular causes over 10 years than among those who received medical therapy alone

265
Q

Trials about Transcatheter MVR in HFrEF px with MR

A

Recently, two large randomized trials of transcatheter MVR using this approach have been conducted in patients with symptomatic HFrEF and moderate-severe functional MR. In the Cardiovascular Outcomes Assessment of the MitraClip Percutaneous Therapy for Heart Failure Patients with Functional Mitral Regurgitation (COAPT) study, patients allocated to MVR versus standard HF therapy experienced a marked reduction in both HF hospitalizations and mortality at 2 years, supporting the efficacy of this approach.

In the second trial, Percutaneous Repair with the MitraClip Device for Severe Functional/Secondary Mitral Regurgitation (MITRA-FR), which employed a similar design, the rates of death or HF hospitalization did not differ between the percutaneous MVR and medical therapy groups. The precise reason for discrepant results between these studies remains unclear but may be related to differences in background utilization of guideline-directed medical therapy, procedural success rates, and patient selection (particularly whether or not the severity of MR is proportionate or disproportionate to the degree of left ventricular cavity dilation).

Because mortality rates at 2 years remain high even with percutaneous MVR, patients with advanced symptoms of HF in whom MR severity is driven principally by end-stage left ventricular remodeling should also be considered for advanced therapies such as mechanical circulatory support.

266
Q

What is the CUPID trial about?

A

Primarily responsible for reincorporating calcium into the sarcoplasmic reticulum during diastole, this target was tested in the CUPID (Efficacy and Safety Study of Genetically Targeted Enzyme Replacement Therapy for Advanced Heart Failure) trial. This study used coronary arterial infusion of adeno-associated virus type 1 carrying the gene for SERCA2a and initially demonstrated that natriuretic peptides were decreased, reverse remodeling was noted, and symptomatic improvements were forthcoming. However, a confirmatory trial failed to meet its primary efficacy endpoint.

267
Q

Histopath of cardiomyopathy caused by anthracyclines

A

Anthracyclines (e.g., doxorubicin) cause characteristic histologic changes of vacuolar degeneration and myofibrillar loss.

268
Q

Deficiency of trace elements such as _____ can cause cardiomyopathy (Keshan’s disease).

A

selenium

269
Q

Diagnostic criteria for Left venticular noncompaction

A

Left ventricular noncompaction is a condition of unknown prevalence that is increasingly suspected with the refinement of imaging techniques. The diagnostic criteria include the presence of multiple trabeculations in the left ventricle distal to the papillary muscles, creating a “spongy” appearance of the apex, but are increasingly recognized as nonspecific findings in other cardiac diseases.

Noncompaction has been associated with multiple genetic variants in the sarcomeric and other genes, such as TAZ (encoding tafazzin)

The three cardinal clinical features of ventricular arrhythmias, embolic events, and heart failure are largely restricted to noncompaction with concomitant systolic dysfunction

270
Q

Most common restrictive cardiomyopathy

A

The most common restrictive cardiomyopathy is amyloidosis, in which a common protein assembles into β-pleated sheets of amyloid fibrils that infiltrate between cells of target organs

Endomyocardial biopsy is virtually 100% reliable for the diagnosis of all amyloid due to the characteristic birefringence pattern of Congo red staining of the amyloid fibrils under polarized light, but immunohistochemistry may be necessary to confirm the amyloid type, as serum or urine electrophoresis may be misleading.

271
Q

Diagnosis of HOCM

A

Cardiac imaging is central to diagnosis, for which the physical examination and ECG are insensitive. The identification of a disease-causing mutation in a proband can focus family evaluations on mutation carriers, but this strategy requires a high degree of certainty that the mutation is truly pathogenic and not a benign DNA variant. Biopsy is not needed to diagnose hypertrophic cardiomyopathy but can be used to exclude infiltrative and metabolic diseases

272
Q

Most commonly affected leaflet in MVP

A

In most patients with MVP, myxomatous degeneration is confined to the mitral valve, although the tricuspid and aortic valves may also be affected. The posterior mitral leaflet is usually more affected than the anterior, and the mitral valve annulus is often dilated.

With posterior leaflet prolapse, the jet of MR is directed anteriorly and the murmur will radiate to the base of the heart.

With anterior leaflet involvement, the jet of MR is directed posteriorly and the murmur will radiate to the axilla and back.

273
Q

T/F

MVP is more common in men

A

False
MVP is more common in women than men and occurs most frequently between the ages of 15 and 30 years; the clinical course is most often benign. MVP may also be observed in older (>50 years) patients, often men, in whom MR is often more severe because of chordal rupture and requires surgical treatment

274
Q

T/F IE profphalxis is indicated in px with MVP

A

Infective endocarditis prophylaxis is indicated for patients with a prior history of endocarditis.

Beta blockers sometimes relieve chest pain and control palpitations

275
Q

MR may occur as a congenital anomaly most commonly as a defect of the .

A

endocardial cushions (atrioventricular cushion defects)

276
Q

Chronic, severe MR is defined by

A

a regurgitant volume ≥60 mL/beat, regurgitant fraction (RF) ≥50%, and effective regurgitant orifice area ≥0.40 cm2

277
Q

Difference in JVP waves in acute vs chronic MR

A

Patients with chronic severe MR, develop marked LA enlargement and increased LA compliance with little if any increase in LA and pulmonary venous pressures for any increase in LA volume. The LA v wave is relatively less prominent

Fatigue, exertional dyspnea, and orthopnea are the most prominent complaints in patients with chronic severe MR

278
Q

Most common cause of mitral stenosis

A

rheumatic fever

279
Q

normal area of MV

A

4-6cm

280
Q

When the mitral valve opening is reduced to <___ cm2 , referred to as “severe” MS, an LA pressure of ~___ mmHg is required to maintain a normal cardiac output (CO).

A

1.5; very severe if <1cm

25

281
Q

In MS and sinus rhythm, the elevated LA and PA wedge pressures exhibit a prominent _____ wave and a gradual pressure decline after the v wave and mitral valve opening (y descent).

A

atrial contraction pattern (a wave)

282
Q

The opening snap (OS) of the mitral valve is most readily audible in _______

A

The opening snap (OS) of the mitral valve is most readily audible in expiration at, or just medial to, the cardiac apex

283
Q

Earliest changes on CXR in px with MS

A

The earliest changes are straightening of the upper left border of the cardiac silhouette, prominence of the main PAs, dilation of the upper lobe pulmonary veins, and posterior displacement of the esophagus by an enlarged LA.

284
Q

Kerley B lines are seen when the resting mean LA pressure exceeds _________

A

Kerley B lines are fine, dense, opaque, horizontal lines that are most prominent in the lower and mid-lung fields that result from distention of interlobular septae and lymphatics with edema when the resting mean LA pressure exceeds ~20 mmHg

285
Q

How do you differentiate ASD vs MS

A

the absence of LA enlargement and of Kerley B lines and the demonstration of fixed splitting of S2 with a grade II or III mid-systolic murmur at the mid to upper left sternal border all favor atrial septal defect over MS

286
Q

How do you differentiate LA myxoma vs MS?

A

patients with an LA myxoma often have features suggestive of a systemic disease, such as weight loss, fever, anemia, systemic emboli, and elevated serum IgG and interleukin 6 (IL-6) concentrations. The auscultatory findings may change markedly with body position. The diagnosis can be established by the demonstration of a characteristic echo-producing mass in the LA with TTE

287
Q

Indications for surgery in MS

A

Unless there is a contraindication, mitral commissurotomy is indicated in symptomatic (New York Heart Association [NYHA] Functional Class II–IV) patients with isolated severe MS, whose effective orifice (valve area) is <~1 cm2 /m2 body surface area, or <1.5 cm2 in normal-sized adults.

Successful commissurotomy is defined by a 50% reduction in the mean mitral valve gradient and a doubling of the mitral valve area

288
Q

Indications for commisurotomy in asymptomatic px with MS

A

unless recurrent systemic embolization or severe pulmonary hypertension has occurred (PA systolic pressures >50 mmHg at rest or >60 mmHg with exercise), commissurotomy is not recommended for patients who are asymptomatic and/or who have mild or moderate stenosis (mitral valve area >1.5 cm2 ).

289
Q

Procedure of choice for MS with significant MR

A

Mitral valve replacement (MVR) is necessary in patients with MS and significant associated MR, those in whom the valve has been severely distorted by previous transcatheter or operative manipulation, or those in whom the surgeon does not find it possible to improve valve function significantly with commissurotomy

290
Q

What conditions do you see AS with AR?

A

The coexistence of hemodynamically significant aortic stenosis (AS) with AR usually excludes all the rarer forms of AR because it occurs almost exclusively in patients with rheumatic or congenital AR

291
Q

In patients with severe AR, _____ sound is usually absent

A

In patients with severe AR, the aortic valve closure sound (A2 ) is usually absent

292
Q

a soft, low-pitched, rumbling mid-to-late diastolic murmur seen in severe AR

A

Austin Flint murmur, a soft, low-pitched, rumbling mid-to-late diastolic murmur

293
Q

ECG findings in chronic severe AR

A

In patients with chronic severe AR, ECG signs of LV hypertrophy are common

In addition, these patients frequently exhibit ST-segment depression and T-wave inversion in leads I, aVL, V5 , and V6 (“LV strain”). Left axis deviation and/or QRS prolongation may also be present

294
Q

Characteristic echo finding in AR in echo

A

A rapid, high-frequency diastolic fluttering of the anterior mitral leaflet produced by the impact of the regurgitant jet is a characteristic finding.

With severe AR, the central jet width assessed by color flow Doppler imaging exceeds 65% of the width of the LV outflow tract, the regurgitant volume is ≥60 mL/beat, the regurgitant fraction is ≥50%, and there is diastolic flow reversal in the proximal portion of the descending thoracic aorta

295
Q

CXR findings in chronic severe AR

A

In chronic severe AR, the apex is displaced downward and to the left in the frontal projection. In the left anterior oblique and lateral projections, the LV is displaced posteriorly and encroaches on the spine

296
Q

Treatment of choice for acute severe AR

A

Surgery is the treatment of choice and is usually necessary within 24 h of diagnosis.

Intra-aortic balloon counterpulsation is contraindicated. Beta blockers are best avoided so as not to reduce the CO further or slow the heart rate, thus allowing more time for diastolic filling of the LV.

297
Q

Indications for surgery for AR

A

Aortic valve replacement (AVR) is indicated for the treatment of severe AR in symptomatic patients irrespective of LV function. In general, the operation should be carried out in asymptomatic patients with severe AR and progressive LV dysfunction defined by an LVEF <55% on serial studies, an LV end-systolic dimension >50 mm (>25 mm/m2 ), or an LV diastolic dimension >65 mm

298
Q

Frequency of monitoring in px with chronic severe AR

A

in patients with chronic severe AR, careful clinical follow-up and noninvasive testing with echocardiography at ~6- to 12-month intervals are necessary if operation is to be undertaken at the optimal time, i.e., after the onset of LV dysfunction but prior to the development of severe symptoms.

Exercise testing may be helpful to assess effort tolerance more objectively. Operation can be deferred as long as the patient both remains asymptomatic and retains normal LV function without severe or progressive chamber dilation

299
Q

T/F

Pulmonic valve stenosis (PS) is essentially a congenital disorder

A

True

With isolated PS, the valve is typically domed. Dysplastic pulmonic valves are seen as part of the Noonan syndrome , which maps to chromosome 12. Mutations in the PTPN1 gene are associated with about half of all cases of Noonan syndrome.

300
Q

Definition of severe PS

A

severe PS is defined by a peak systolic gradient across the pulmonic valve of >64 mmHg (mean gradient >35 mm Hg, Doppler jet velocity >4 m/s);

300
Q

CXR findings in severe PS

A

Chest x-ray findings include poststenotic dilation of the main PA in the frontal plane projection and filling of the retrosternal airspace due to RV enlargement on the lateral film

301
Q

A mean diastolic pressure gradient of ___mmHg in TS is usually sufficient to elevate the mean RA pressure to levels that result in systemic venous congestion

A

A diastolic pressure gradient between the right atrium (RA) and right ventricle (RV) defines TS. It is augmented when the transvalvular blood flow increases during inspiration and declines during expiration. A mean diastolic pressure gradient of 4 mmHg is usually sufficient to elevate the mean RA pressure to levels that result in systemic venous congestion.

302
Q

Characteristic symptoms of TS

A

Characteristically, patients with severe TS complain of relatively little dyspnea for the degree of hepatomegaly, ascites, and edema that they have. However, fatigue secondary to a low CO and discomfort due to refractory edema, ascites, and marked hepatomegaly are common in patients with advanced TS and/or TR.

302
Q

JVP waves seen in severe TS

A

The jugular veins are distended, and in patients with sinus rhythm, there may be giant a waves. The v waves are less conspicuous, and because tricuspid obstruction impedes RA emptying during diastole, there is a slow y descent.

303
Q

Definition of severe TS

A

Severe TS is characterized by a valve area ≤1 cm2 or pressure half-time of ≥190 ms.

303
Q

ECG findings in TS

A

The electrocardiogram (ECG) features of RA enlargement include tall, peaked P waves in lead II, as well as prominent, upright P waves in lead V1 .

The absence of ECG evidence of RV hypertrophy (RVH) in a patient with right-sided heart failure who is believed to have MS should suggest associated tricuspid valve disease

304
Q

ECG changes in TR

A

The ECG may show changes characteristic of the lesion responsible for the TR, e.g., an inferior Q-wave MI suggestive of a prior RV MI, RVH, or a bizarre right bundle branch block–type pattern with preexcitation in patients with Ebstein’s anomaly. ECG signs of RA enlargement may be present in patients with sinus rhythm; AF is frequently noted.

305
Q

Indications for surgery in px with TR

A

Tricuspid valve surgery is recommended for patients with severe TR who are undergoing left-sided valve surgery and is also undertaken frequently for treatment of even moderate TR in patients undergoing left-sided valve surgery who have tricuspid annular dilation (>40 mm), a history of right heart failure, or PA hypertension.

306
Q

most common valve lesion among adult patients with chronic valvular heart disease

A

AS

most common valve lesion among adult patients with chronic valvular heart disease; the majority of adult patients with symptomatic, valvular AS are male

307
Q

Bicuspid aortic valve disease

A

most common congenital heart valve defect

307
Q

AS is rarely of clinical importance until the valve orifice has narrowed to _____

A

~1 cm2

307
Q

Genetic polymorphisms associated with AS

A

Genetic polymorphisms involving the vitamin D receptor, the estrogen receptor in postmenopausal women, interleukin 10, and apolipoprotein E4 have been linked to the development of calcific AS, and a strong familial clustering of cases with trileaflet valves has been reported from western France.

308
Q

A double apical impulse (with a palpable S4 ) may be appreciated in px withAS, particularly with the patient in the ___________ position.

A

A double apical impulse (with a palpable S4 ) may be appreciated, particularly with the patient in the left lateral recumbent position.

309
Q

Auscultatory findings in px with AS

A

An early systolic ejection sound is frequently audible in children, adolescents, and young adults with congenital BAV disease. The murmur of AS is described as an ejection (mid) systolic murmur that commences shortly after the S1 , increases in intensity to reach a peak toward the middle of ejection, and ends just before aortic valve closure

This sound usually disappears when the valve becomes calcified and rigid.

As AS increases in severity, LV systole may become prolonged so that the aortic valve closure sound no longer precedes the pulmonic valve closure sound, and the two components may become synchronous, or aortic valve closure may even follow pulmonic valve closure, causing paradoxical splitting of S2.

The sound of aortic valve closure can be heard most frequently in patients with AS who have pliable valves, and calcification diminishes the intensity of this sound.

Frequently, an S4 is audible at the apex and reflects the presence of LV hypertrophy and an elevated LV end-diastolic pressure; an S3 generally occurs late in the course, when the LV dilates and its systolic function becomes severely compromised.

310
Q

ECG changes in AS

A

In most patients with severe AS, there is LV hypertrophy. In advanced cases, ST-segment depression and T-wave inversion (LV “strain”) in standard leads I and aVL and in the left precordial leads are evident. However, there is no close correlation between the ECG and the hemodynamic severity of obstruction, and the absence of ECG signs of LV hypertrophy does not exclude severe obstruction

311
Q

Patients with severe AS (i.e., valve area <1 cm2 ) with a relatively low mean gradient (<____mmHg) despite a normal EF (low-flow, lowgradient, severe AS with normal EF) are often hypertensive, and efforts to control their systemic blood pressure should be optimized before Doppler echocardiography is repeated

A

40

Dobutamine stress echocardiography is useful for the evaluation of patients with AS and severe LV systolic dysfunction (low-flow, low-gradient, severe AS with reduced EF), in whom the severity of the AS can often be difficult to judge.

312
Q

T/F Statins and RAAS inhibitors are beneficial in px with AS

A

Neither HMG-CoA reductase inhibitors (“statins”) nor inhibitors of the renin-angiotensin-aldosterone system slow the rate of progression of AS. The use of statin medications should be driven by considerations regarding primary and secondary prevention of atherosclerotic cardiovascular disease (ASCVD) events.

The need for endocarditis prophylaxis is restricted to AS patients with a prior history of endocarditis

313
Q

Surgical indication for AS

A

Operation is indicated in patients with severe AS (valve area <1 cm2 or 0.6 cm2 /m2 body surface area) who are symptomatic, those who exhibit LV systolic dysfunction (EF <50%), and those with AS due to BAV disease and an aneurysmal root or ascending aorta (maximal dimension >5.5 cm).

Operation should be carried out promptly (1–3 months) after symptom onset

314
Q

Factors which favor SAVR vs TAVI

A
315
Q

Disorder with pericardial knock and pericardial calcification

A

constrictive pericarditis

316
Q

Most frequent complication of viral or idiopathic pericarditis

A

The most frequent complication is recurrent (relapsing) pericarditis, which occurs in about one-fourth of patients with acute idiopathic pericarditis.

317
Q

This disorder results when the healing of an acute fibrinous or serofibrinous pericarditis or the resorption of a chronic pericardial effusion is followed by obliteration of the pericardial cavity with the formation of granulation tissue

A

Chronic constrictive pericarditis

318
Q

Pericardial calcification is most common in ________ pericarditis.

A

Pericardial calcification is most common in tuberculous pericarditis.

319
Q

only definitive treatment of constrictive pericarditis

A

Pericardial resection is the only definitive treatment of constrictive pericarditis and should be as complete as possible.

320
Q

This form of pericardial disease is characterized by the combination of a tense effusion in the pericardial space and constriction of the heart by thickened pericardium

A

Subacute Effusive Constrictive Pericarditis

321
Q

If the biopsy specimen shows a thickened pericardium after 2–4 weeks of antituberculous therapy, _____ should be performed

A

pericardiectomy should be performed to prevent the development of constriction. Tubercular cardiac constriction should be treated surgically while the patient is receiving antituberculous chemotherapy.

322
Q

A useful finding in assessing a patient with chest discomfort is the fact that myocardial ischemic discomfort does not radiate to the trapezius muscles; that radiation pattern is more typical of ________

A

pericarditis

323
Q

The major benefit of high-sensitivity CRP in IHD

A

The major benefit of high-sensitivity CRP is in reclassifying the risk of IHD in patients in the “intermediate” risk category on the basis of traditional risk factors.

324
Q

In stress test Exercise duration is usually symptom-limited, and the test is discontinued upon evidence of chest discomfort, severe shortness of breath, dizziness, severe fatigue, ST-segment depression >_______, a fall in systolic blood pressure >10 mmHg, or the development of a ventricular tachyarrhythmia

A

0.2 mV (2 mm)

325
Q

In stress test The ischemic ST-segment response generally is defined as flat or downsloping depression of the ST segment >____ below baseline (i.e., the PR segment) and lasting longer than _____

A

0.1 mV

0.08 s

so ischemic response 0.1 then 0.2 stop na

Upsloping or junctional ST-segment changes are not considered characteristic of ischemia and do not constitute a positive test.

326
Q

In interpreting ECG stress tests, the probability that coronary artery disease (CAD) exists in the patient or population under study (i.e., pretest probability) should be considered. A positive result on exercise indicates that the likelihood of CAD is ____% in males who are >50 years with a history of typical angina pectoris and who develop chest discomfort during the test.

A

98%

327
Q

Contraindications to exercise stress testing include

A

rest angina within 48 h, unstable rhythm, severe aortic stenosis, acute myocarditis, uncontrolled heart failure, severe pulmonary hypertension, and active infective endocarditis

328
Q

Characteristics/ factors in stress testing that increase the specificity of the test and suggests severe IHD and a high risk of future adverse events

A

The development of angina and/or severe (>0.2 mV) ST-segment depression at a low workload, i.e., before completion of stage II of the Bruce protocol, and/or STsegment depression that persists >5 min after the termination of exercise increases the specificity of the test and suggests severe IHD and a high risk of future adverse events

329
Q

On cardiac catheterization,_____________ are the most important signs of LV dysfunction and are associated with a poor prognosis

A

On cardiac catheterization, elevations of LV end-diastolic pressure and ventricular volume and reduced ejection fraction are the most important signs of LV dysfunction and are associated with a poor prognosis

330
Q

Activities requiring 5-7 METS

A
331
Q

What CCB has constipation as side effect?

A

nifedipine

332
Q

To minimize the effects of nitrate tolerance, the minimum effective dose should be used and a minimum of 8 h each day kept free of the drug to restore any useful response

A

8

332
Q

Since sudden discontinuation of beta blockers can intensify ischemia, the doses should be tapered over __ weeks

A

2ni

333
Q

T/F Stroke rates are lower in PCI compared to CABG

A

True

334
Q

In patients with NSTEMI, there is a characteristic temporal rise of the plasma concentration, peaking at ____ h after onset of symptoms and gradually decreasing thereafter

A

12-24h

The 1-h rapid rule-out MI algorithm (no abnormal elevation of hsTn at 0 or 1 hour after presentation) has been recommended by recent practice guidelines

335
Q

Heart rate–slowing calcium channel blockers, e.g., verapamil or diltiazem, are recommended for patients _____________-

A

who have persistent symptoms or ECG signs of ischemia after treatment with full-dose nitrates and beta blockers and in patients with contraindications to either class of these agents

336
Q

Reversible P2Y12 inhibitor

A

Ticagrelor

337
Q

DAPT should continue for at least ______ months in patients with NSTE-ACS without an indication for long-term full-dose anticoagulation; the duration of DAPT is dependent upon the risk of bleeding versus thrombosis

A

3 months (preferably 12 months)

In patients with atrial fibrillation (including patients with NSTE-ACS) treated with an oral anticoagulant who undergo PCI, the duration of DAPT should be shortened (e.g., stop aspirin after hospital discharge or up to 4 weeks post PCI, except in patients at very high risk for ischemic events), and continue P2Y12 inhibitor plus DOAC for 1 year.

338
Q

Diagnostic hallmark of prinzmetal variant angina

A

Coronary angiography demonstrates transient coronary spasm as the diagnostic hallmark of PVA

339
Q

In PCI The first-generation devices were coated with either _____.

Second-generation drug-eluting stents use newer agents such as ______

A

1st gen- sirolimus or paclitaxel

2nd gen- everolimus, biolimus, and zotarolimus

340
Q

The most common reason to decide not to do PCI is that

A

lesion(s) felt to be responsible for the patient’s symptoms is not treatable. This is most commonly due to the presence of a chronic total occlusion (>3 months in duration) with unfavorable characteristics

Lesions with the highest success are called type A lesions (such as proximal noncalcified subtotal lesions), and those with the lowest success or highest complication rate are type C lesions (such as chronic total occlusions). Intermediate lesions are classified as type B1 or B2 depending on the number of unfavorable characteristics.

341
Q

trial that supports complete revascularization of nonculprit lesions in STEMI either in the hospital or in the few weeks after discharge

A

The Complete Versus Culprit-Only Revascularization Strategies to Treat Multivessel Disease After Early PCI for STEMI (COMPLETE) trial supports complete revascularization of nonculprit lesions in STEMI either in the hospital or in the few weeks after discharge

342
Q

Temperature elevations up to 38°C may be observed during the _____ after STEMI

A

first week

343
Q

In STEMI, The arterial pressure is variable; in most patients with transmural infarction, systolic pressure declines by ~_______ mmHg from the preinfarction state.

A

10–15 mmHg

344
Q

Most out-ofhospital deaths from STEMI result from

A

sudden development of ventricular fibrillation

The vast majority of deaths due to ventricular fibrillation occur within the first 24 h of the onset of symptoms, and of these, over half occur in the first hour.

345
Q

Morphine is a very effective analgesic for the pain associated with STEMI. However, it may reduce sympathetically mediated arteriolar and venous constriction, and the resulting venous pooling may reduce cardiac output and arterial pressure. These hemodynamic disturbances usually respond promptly to

A

elevation of the legs, but in some patients, volume expansion with intravenous saline is required.

346
Q

Morphine also has a vagotonic effect and may cause bradycardia or advanced degrees of heart block, particularly in patients with inferior infarction. These side effects usually respond to

A

atropine (0.5 mg intravenously).

347
Q

T/F NSTEMI px will not benefit from fibrinolysis

A

True

In the absence of ST-segment elevation, fibrinolysis is not helpful, and evidence exists suggesting that it may be harmful

348
Q

Primary and secondary goal of antiplt and anticoag in px with STEMI

A

The primary goal of treatment with antiplatelet and anticoagulant agents is to maintain patency of the infarct-related artery, in conjunction with reperfusion strategies. A secondary goal is to reduce the patient’s tendency to thrombosis and, thus, the likelihood of mural thrombus formation or deep-venous thrombosis. The degree to which antiplatelet and anticoagulant therapy achieves these goals partly determines how effectively it reduces the risk of mortality from STEMI.

348
Q

Compared with fibrinolysis, primary PCI is generally preferred when

A

the diagnosis is in doubt, cardiogenic shock is present, bleeding risk is increased, or symptoms have been present for at least 2–3 h when the clot is more mature and less easily lysed by fibrinolytic drugs.

However, PCI is expensive in terms of personnel and facilities, and its applicability is limited by its availability, around the clock, in only a minority of hospitals

349
Q

Why should fondaparinux not to be used alone for px undergoing coronary angio and PCI

A

Owing to the risk of catheter thrombosis, fondaparinux should not be used alone at the time of coronary angiography and PCI but should be combined with another anticoagulant with antithrombin activity such as UFH or bivalirudin.

350
Q

Patients with an anterior location of the infarction, severe LV dysfunction, heart failure, a history of embolism, two-dimensional echocardiographic evidence of mural thrombus, or atrial fibrillation are at increased risk of systemic or pulmonary thromboembolism. Such individuals should receive full therapeutic levels of anticoagulant therapy (LMWH or UFH) while hospitalized, followed by at least 3 months of _______

A

warfarin therapy.

351
Q

Hemodynamic evidence of abnormal global LV function appears when contraction is seriously impaired in ___% of the left ventricle

A

20–25

352
Q

Infarction of ≥___% of the left ventricle usually results in cardiogenic shock

A

40

353
Q

hypokalemia and hypomagnesemia are risk factors for ventricular fibrillation in patients with STEMI; to reduce the risk, the serum potassium concentration should be adjusted to ~___ mmol/L and magnesium to ~___ mmol/L.

A

4.5

2.0

354
Q

usually the treatment of choice for supraventricular arrhythmias if heart failure is present

A

Digoxin

as per harrisons

If heart failure is absent, beta blockers, verapamil, and diltiazem are suitable alternatives for controlling the ventricular rate, as they may also help to control ischemia. If the abnormal rhythm persists for >2 h with a ventricular rate >120 beats/min or if tachycardia induces heart failure, shock, or ischemia (as manifested by recurrent pain or ECG changes), a synchronized electroshock (100–200 J monophasic waveform) should be used.

355
Q

Two-dimensional echocardiography reveals LV thrombi in about onethird of patients with ______ wall infarction

A

anterior

but in few patients with inferior or posterior infarction

356
Q

The usual duration of hospitalization for an uncomplicated STEMI is about _______

A

3–5 days.

357
Q

leading cause of PAD in patients >40 years old.

A

Atherosclerosis is the leading cause of PAD in patients >40 years old.

The primary sites of involvement are the abdominal aorta and iliac arteries (30% of symptomatic patients), the femoral and popliteal arteries (80–90% of patients), and the more distal vessels, including the tibial and peroneal arteries (40–50% of patients)

358
Q

T/F

DAPT is beneficial in px with PAD

A

The benefit of dual antiplatelet therapy with both aspirin and clopidogrel compared with aspirin alone in reducing cardiovascular morbidity and mortality rates in patients with PAD is uncertain

When added to other antiplatelet therapy, vorapaxar, a protease activated receptor-1 antagonist that inhibits thrombinmediated platelet activation, decreases the risk of adverse cardiovascular events in patients with atherosclerosis, including PAD. It also reduces the risk of acute limb ischemia and peripheral revascularization;

358
Q

How long do you do supervised exercise training for px with PAD?

A

Supervised exercise training programs for 30- to 45-min sessions, at least three per week for 12 weeks, prolong walking distance

359
Q

Intraarterial thrombolytic therapy with recombinant tissue plasminogen activator, reteplase, or tenecteplase is most effective when acute arterial occlusion is recent (<__ weeks) and caused by a thrombus in an atherosclerotic vessel, arterial bypass graft, or occluded stent.

A

2

360
Q

Surgical revascularization is preferred when restoration of blood flow must occur within __ h to prevent limb loss or when symptoms of occlusion have been present for >__ weeks.

A

24h

2 wk

361
Q

Longest vein in the body

A

The great saphenous vein is the longest vein in the body. It originates on the medial side of the foot and ascends anterior to the medial malleolus and then along the medial side of the calf and thigh, and drains into the common femoral vein

362
Q

Varicose veins are dilated, bulging, tortuous superficial veins, measuring at least __ mm in diameter.

A

Varicose veins are dilated, bulging, tortuous superficial veins, measuring at least 3 mm in diameter.

363
Q

How do you perform Brodie-Trendelenburg test ?

A

The Brodie-Trendelenburg test is used to determine whether varicose veins are secondary to deep-venous insufficiency. As the patient is lying supine, the leg is elevated and the veins allowed to empty. Then, a tourniquet is placed on the proximal part of the thigh and the patient is asked to stand. Filling of the varicose veins within 30 s indicates that the varicose veins are caused by deep-venous insufficiency and incompetent perforating veins. Primary varicose veins with superficial venous insufficiency are the likely diagnosis if venous refilling occurs promptly after tourniquet removal.

363
Q

How do you peform the Perthes test?

A

The Perthes test assesses the possibility of deep-venous obstruction. A tourniquet is placed on the midthigh after the patient has stood, and the varicose veins are filled. The patient is then instructed to walk for 5 min. A patent deep-venous system and competent perforating veins enable the superficial veins below the tourniquet to collapse. Deep-venous obstruction is likely to be present if the superficial veins distend further with walking

364
Q

How do you classify CVI based on clinical classification

A
365
Q

Aside from compression bandages, what other conservative management can be offered to px with CVI?

A

In addition to a compression bandage or stocking, patients with venous ulcers also may be treated with low-adherent absorbent dressings that take up exudates while maintaining a moist environment.

Ulcers should be debrided of necrotic tissue. Antibiotics are not indicated unless the ulcer is infected. The multilayered compression bandage or graduated compression garment is then put over the dressing.

Others: sclerotherapy, non thermal ablation

365
Q

Based on CPG 2020, how do you classify hypertension among adult Filipinoes

A

Observational studies have shown that for every 10 mmHg increase in the blood pressure from 115/75mmHg, there is doubling of cardiovascular risk thus, the rationale of setting the BP of < 120/80mmHg as the optimal level at which cardiovascular risk is set at the minimum.

366
Q

Correct cuff size for adult Filipinos

A

The cuff should be of correct size, placed snug over the upper arm that is preferably without sleeves, with its middle portion at the level of the heart. It should cover 40% of the upper arm and 80% of the arm circumference (standard bladder for adults is 13 cm wide, 22-24 cm long)

367
Q

Proper method of measuring BP

A

> When using a manual device, cuff deflation should be done at 2 mmHg/sec.
Actual auscultatory determination is then done by inflating the cuff 20-30 mmHg above this palpated estimate.
For auscultatory determination, use the fifth Korotkoff sound to determine the diastolic BP. ●
Use an average ≥ 2 readings obtained on ≥ 2 occasions as an estimate of the BP level. ● The interval between BP measurements should be 1-2 min apart

368
Q

Definition of hypertension based on ABPM vs HBPM

A
369
Q

BP targets among Adult Filipinos

A

A therapeutic threshold of greater than or equal to 140/90 mmHg to achieve a goal of less than 130/80 is recommended for all patients with hypertension.

For the very elderly, defined as 80 years old and above, a therapeutic threshold of greater than or equal to 150/90 to achieve a goal BP of less than 140/90 is recommended

370
Q

Laboratory tests to request in hypertensive px

A

> complete blood count, sodium, potassium, creatinine and blood urea nitrogen
fasting glucose and lipid profile.
To check for albuminuria, it is recommended to do a dipstick urinalysis, or if available, urine micral test, spot or 24-hour urine albumin-creatinine ratio.
To check for LVH 12L ECG or echocardiography can be done.
For cardiac dysfunction, atrial dilation or aortic coarctation, echocardiography is recommended.
A carotid ultrasound is requested to check for atherosclerosis or stenosis.
A simple fundoscopy can be utilized to check for retinal changes, hemorrhages, and papilledema.

> Additional tests can be done to check for other HMOD or secondary causes such as a cranial CT/MRI, ankle-brachial index, serum uric acid, thyroid-stimulating hormone, aldosterone-renin ratio, serum and urinary metanephrines, salivary or serum cortisol, and liver function tests

371
Q

Lifestyle modifications advised to hypertensive px as per CPG

A

Sodium restriction to as low as 1500 mg/day Dietary Approaches to Stop Hypertension (DASH) meal plan
Aerobic physical activity and (dynamic) resistance exercises
Abstinence from alcohol or moderate alcohol intake
Significant weight loss of > 5% of the baseline weight
Smoking cessation

**should be tried for 3 months, if BP target not achieved, start anti hpn meds

372
Q

Aerobic physical activity and exercises reduce blood pressure by around ______ with the benefit being greatest for those who are sedentary.

A

Aerobic physical activity and exercises reduce blood pressure by around 8.3/5.2 mm Hg with the benefit being greatest for those who are sedentary.

373
Q

Beta blockers are suitable as initial therapy in hypertensive patients with

A

Beta blockers are suitable as initial therapy in hypertensive patients with coronary artery disease, acute coronary syndrome, high sympathetic drive, and pregnant women. We prefer the newer generation beta blockers for those with congestive heart failure such as bisoprolol, carvedilol, metoprolol succinate or nebivolol.

374
Q

What BP should you start dual anti hpn meds?

A

Among patients with BP >150/100 mm Hg (or >160/100 mm Hg in the elderly), a combination of 2 agents, preferably combination of a RAAS inhibitor (ARB/ACE-is) and CCB or diuretic, should be given initially since it is unlikely that any single agent would be sufficient to achieve the BP target.

375
Q

Both thiazides and thiazide-like agents are less effective antihypertensive agents in patients with a reduced GFR (eGFR <__mL/min). Loop diuretics such as furosemide (or torsemide) should be used for its BP lowering effect when the eGFR is <__ mL/min.

A

Both thiazides and thiazide-like agents are less effective antihypertensive agents in patients with a reduced GFR (eGFR <45 mL/min). Loop diuretics such as furosemide (or torsemide) should be used for its BP lowering effect when the eGFR is <30 mL/min.

376
Q

Based on CPG 2020, Among persons with diabetes and hypertension, it is recommended that drug therapy (along with lifestyle advice) be initiated at a blood pressure of _______

A

Among persons with diabetes and hypertension, it is recommended that drug therapy (along with lifestyle advice) be initiated at a blood pressure of 140/90 mm Hg or higher

377
Q

BP target in px with hypertension and T2DM

A

A blood pressure target of <130/80 mm Hg is recommended for most persons with diabetes mellitus and hypertension; however, do not lower down the blood pressure below 120/70 due to an increased risk for cardiovascular events.

378
Q

Among adult patients with CKD who are pre-dialysis, what is the target blood pressure to prevent cardiovascular complications and renal progression?

A

For routine office blood pressure measurement, maintain a BP target consistently less than 140 mmHg systolic and less than 90 mmHg diastolic in patients with low risk of cardiovascular disease and CKD grade 4 and 5, or if with adverse effect on intensive target of less than 130/80 mmHg. CKD patients with high cardiovascular risk or CKD grade 3 or earlier is recommended to have a blood pressure target of less than 130/80 mmHg

A systolic BP of less than 120 mmHg using a standardized office BP measurement is targeted, when tolerated, among adults with high BP and non-dialysis CKD

379
Q

Among adult patients with CKD, what is the level of blood pressure to start initiation with two antihypertensive drugs to prevent cardiovascular complications and renal progression?

A

Patients with confirmed office-based blood pressure of more than or equal to 160/100 mmHg should, in addition to lifestyle modification, have prompt initiation and timely titration of two drugs or a single-pill combination of drugs demonstrated to reduce cardiovascular events

380
Q

Among adult patients with CKD with albuminuria/proteinuria, what is the anti-hypertensive of choice to prevent cardiovascular complications and renal progression?

A

An ACE inhibitor or Angiotensin receptor blocker, at maximally tolerated dose is the recommended first-line treatment for hypertension in CKD patients with urinary albumin-tocreatinine ratio more than or equal to 30 mg/g (or equivalent)

381
Q

For adults with acute ischemic stroke (AIS) who are eligible for intravenous (IV) thrombolysis but not for mechanical thrombectomy, what is the threshold for pharmacological treatment and the target blood pressure (BP)?

A

For adults with AIS who are eligible for IV thrombolysis but not for mechanical thrombectomy, it is recommended that the BP be maintained <185/110 mmHg prior to treatment and during infusion. For the next 24 hours after treatment, the BP is recommended to be maintained <180/105 mmHg.

the recommended pharmacologic agent is Nicardipine 5mg/hr IV, titrate up by 2.5mg/hr every 5-15 minutes, with maximum of 15mg/hr. If available, labetalol 10 mg IV over 1-2 minutes followed by continuous IV infusion of 2-8 mg/min may also be used

382
Q

For adults with AIS who are eligible for mechanical thrombectomy (with or without IV thrombolysis), what is the threshold for pharmacological treatment and the target BP?

A

In patients for whom mechanical thrombectomy is planned, it is reasonable to maintain BP <185/110 mmHg before, during, and after the procedure. After the procedure, the target BP depends on the recanalization status, with lower BP thresholds recommended for those with successful recanalization and reperfusion

383
Q

In patients for whom mechanical thrombectomy is planned, it is reasonable to maintain BP <185/110 mmHg before, during, and after the procedure. After the procedure, the target BP depends on the recanalization status, with lower BP thresholds recommended for those with successful recanalization and reperfusion

A

For adult patients with acute ICH, the threshold for BP lowering is SBP ≥ 180 mmHg.

The target SBP is <180 mm Hg. In patients with SBP ≥180 mm Hg, careful BP lowering to 140 to 160 mm Hg should be considered. The magnitude of BP reduction is dependent on the clinical context. It should be careful SBP lowering (avoiding reductions ≥60mmHg in 1 h). It is also recommended to keep the blood pressure stable and avoid variability. Likewise, it is also recommended not to lower the BP acutely to <140mmHg

In the ACC/AHA guidelines, adults with ICH who present with SBP >220 mmHg, it is reasonable to use continuous IV drug infusion and close BP monitoring to lower SBP.

384
Q

MAP for acute ischemic stroke

A

A MAP of 110 to 130 mmHg must be maintained to improve cerebral perfusion pressure.

385
Q

For adults who have a history of stroke, what is the target blood pressure level for secondary prevention?

A

For adults with history of stroke, the target blood pressure level for secondary prevention is less than or equal to 130/80 mm Hg.

RAS blockers, CCBs and thiazide diuretics remain to be the first-line pharmacologic agents in hypertension management for secondary stroke prevention.

386
Q

Definition of proteinuria in pregnancy

A

300 mg or more per 24 hour urine collection (or this amount extrapolated from a timed collection)
Protein/creatinine ratio of 0.3 mg/dl or more or Dipstick reading of 2+ (used only if other quantitative methods not available)

387
Q

Definition of chronic hypertension in pregnancy

A

Hypertension of any cause that predates pregnancy.
BP > 140/90 mm Hg before pregnancy or before 20 weeks gestation or both.

388
Q

Definition of preeclampsia

A
389
Q

Definition of severe preeclampsia

A
390
Q

What antihypertensive agents can be used for urgent blood pressure control in pregnancy?

A

Acute-onset severe hypertension (systolic BP of 160 mm Hg or more or diastolic BP of 110 mm Hg or more, or both) can occur in the prenatal, intrapartum and postpartum period. It is accurately measured using standard techniques and is persistent for 15 minutes or more. The first line of treatment is intravenous (IV) hydralazine and labetalol; intravenous nicardipine is also an option. Extended release oral nifedipine also may be considered as a first line therapy, particularly when IV access is not available. Use of these drugs does not require cardiac monitoring

Although parenteral antihypertensive therapy may be needed initially for acute control of blood pressure, oral medications can be used as expectant management is continued. One approach is to begin an initial regimen of labetalol at 200 mg orally every 12 hours and increase the dose up to 800 mg orally every 8–12 hours as needed (maximum total 2,400 mg/d). If the maximum dose is inadequate to achieve the desired blood pressure goal, or the dosage is limited by adverse effect, then short-acting oral nifedipine can be added gradually.

391
Q

An ankle-brachial index <____ is associated with elevated blood pressure, particularly systolic blood pressure.

A

An ankle-brachial index <0.80 is associated with elevated blood pressure, particularly systolic blood pressure.

392
Q

T/F Nighttime blood pressures are blower than daytime

A

True
Nighttime blood pressures are generally 10–20% lower than daytime blood pressures, and an attenuated nighttime blood pressure “dip” is associated with increased cardiovascular disease risk.

393
Q

gold standard for evaluation and identification of renal artery lesions

A

Contrast arteriography remains the “gold standard” for evaluation and identification of renal artery lesions

Functionally significant lesions generally occlude >70% of the lumen of the affected renal artery. On angiography, the presence of collateral vessels to the ischemic kidney suggests a functionally significant lesion. A lateralizing renal vein renin ratio (ratio >1.5 of affected side/contralateral side) has a 90% predictive value for a lesion that would respond to vascular repair; however, the false-negative rate for blood pressure control is 50–60%.

394
Q

most effective medical therapies for renovascular hypertension

A

The most effective medical therapies for renovascular hypertension include an ACE inhibitor or an angiotensin II receptor blocker; however, these agents decrease glomerular filtration rate in a stenotic kidney owing to efferent renal arteriolar dilation. In the presence of bilateral renal artery stenosis or renal artery stenosis to a solitary kidney, progressive renal insufficiency may result from the use of these agents

395
Q

The ratio of plasma aldosterone (PA) to PRA (PA/PRA) is a useful screening test for primary hyperaldosteronism . A ratio >____ in conjunction with a PA concentration >____pmol/L (>20 ng/dL) reportedly has a sensitivity of 90% and a specificity of 91% for an aldosterone-producing adenoma.

A

These measurements preferably are obtained in ambulatory patients in the morning

> 30:1

555

Current recommendations are to withdraw aldosterone antagonists for at least 4–6 weeks before obtaining these measurements.

396
Q

the definitive treatment of pheochromocytoma and results in cure in ~90% of patients

A

Surgical excision

397
Q

What are the effects of RAAS blockers on glucose metabolism?

A

ACEIs and ARBs improve insulin action and ameliorate the adverse effects of diuretics on glucose metabolism.

398
Q

Parenteral medications for hypertensive emergencies

A
399
Q

Based on CPG 2020 For individuals without diabetes aged ≥ __ years with LDL-C ≥ ___ mg/dL AND ≥ 2 risk factors*, without atherosclerotic cardiovascular disease, statins are RECOMMENDED for the prevention of cardiovascular events.

A

For individuals without diabetes aged ≥ 45 years with LDL-C ≥ 130 mg/dL AND ≥ 2 risk factors*, without atherosclerotic cardiovascular disease, statins are RECOMMENDED for the prevention of cardiovascular events.

Among chronic kidney disease individuals not on dialysis (CKD-ND), statins are RECOMMENDED for the prevention of cardiovascular events.

400
Q

Among individuals with diabetes, routinely adding fibrates on top of statin therapy is not recommended for primary or secondary prevention of cardiovascular disease

However, adding fibrates to statins may be considered among men with controlled diabetes, low HDL-C (< ___ mg/dl) and persistently high triglycerides (>___ mg/dl) for prevention of CV disease

A

Among individuals with diabetes, routinely adding fibrates on top of statin therapy is not recommended for primary or secondary prevention of cardiovascular disease

However, adding fibrates to statins may be considered among men with controlled diabetes, low HDL-C (<35 mg/dl) and persistently high triglycerides (>200 mg/dl) for prevention of CV disease

401
Q

Based on CPG 2020 Among individuals with ASCVD on statin therapy at goal LDL-C, but with persistently high triglyceride levels of ____ mg/dl, omega fatty acids (pure EPA) MAY be given

A

Among individuals with ASCVD on statin therapy at goal LDL-C, but with persistently high triglyceride levels of 135-499 mg/dl, omega fatty acids (pure EPA) MAY be given

402
Q

Based on CPG 2020, which set of patients may be advised intermittent fasting as weight loss strategy

A

In this light, similar to the consensus statement, IF (i.e. alternate day fasting or modified fasting regimens) may be used as a weight loss strategy in obese adult individuals without established ASCVD for a period of 6 to 12 months.

However, because of lack of evidence, the guidelines do not recommend IF for diabetic individuals and in individuals with clinical ASCVD for ASCVD protection

403
Q

Diagnosis of familial hypercholesterolemia

A
404
Q

Among individuals, can fibrates be given on top of statin therapy once LDL-C goal is not achieved?

A

Not recommended

However, adding fibrates to statins may be considered among men with controlled diabetes and low HDL-C (<35 mg/dl) and persistently high triglycerides (>200 mg/dl) for additional prevention of CV disease

405
Q

Target LDL based on CPG 2020

A
406
Q

Based on CPG 2020, when should you hold statins?

A
407
Q

Based on CPG 2020, how long should you advise statin holiday if the px develops rhabdomyolysis?

A

6 weeks

408
Q

Virchows triad

A

Virchow’s triad of venous stasis, hypercoagulability, and endothelial injury leads to recruitment of activated platelets, which release microparticles

409
Q

Definition of submassive PE

A

Submassive (intermediate-risk) PE accounts for 20–25% of patients and is characterized by RV dysfunction despite normal systemic arterial
pressure.

410
Q

Upper limit of D dimer

A

The standard upper of limit of a d-dimer is 500 ng/mL. However, guidelines now recommend use of an age-adjusted d-dimer when ruling out acute PE. The age-adjusted d-dimer applies to patients older than 50 years of age with low or intermediate clinical probability of PE. To calculate the upper limit of normal d-dimer in these patients, multiply the age by 10.

** used only for PE not DVT

411
Q

What constitutes Wells score?

A
412
Q

Definition of high probability scan for PE

A

A high-probability scan for PE is defined as two or more segmental perfusion defects in the presence of normal ventilation.

The diagnosis of PE is very unlikely in patients with normal and nearly normal scans and, in contrast, is ~90% certain in patients with high-probability scans

413
Q

What constitutes primary therapy for DVT?

A

Primary therapy consists of clot dissolution with pharmacomechanical therapy using low-dose catheter-directed thrombolysis.

414
Q

Secondary prevention of DVT

A

Anticoagulation or placement of an inferior vena cava (IVC) filter constitutes secondary prevention of VTE.

IVC filters are indicated in patients with an absolute contraindication to anticoagulation and for those who have suffered recurrent VTE while receiving therapeutic doses of anticoagulation.

415
Q

When should you replace compression stockings for DVT?

A

3 months

416
Q

Target APTT for UFH for VTE

A

UFH is dosed to achieve a target activated partial thromboplastin time (aPTT) of 60–80 s.

Use an initial bolus of 80 U/kg, followed by an initial infusion rate of 18 U/kg per h in patients with normal liver function.

417
Q

Anticoags that may be used for VTE

A
418
Q

The full effect of warfarin requires daily therapy for at least ___ days

A

The full effect of warfarin requires daily therapy for at least 5 days

Overlapping UFH, LMWH, fondaparinux, or parenteral direct thrombin inhibitors with warfarin for at least 5 days will nullify the early procoagulant effect of warfarin. The dose of warfarin is usually targeted to achieve a target international normalized ratio (INR) of 2.5, with a range of 2.0–3.0

419
Q

Management of massive PE

A

For patients with massive PE and hypotension, replete volume with 500 mL of normal saline. Additional fluid should be infused with extreme caution because excessive fluid administration exacerbates RV wall stress, causes more profound RV ischemia, and worsens LV compliance and filling by causing further interventricular septal shift toward the LV. Norepinephrine and dobutamine are first line vasopressor and inotropic agents, respectively, for treatment of PE-related shock.

Norepinephrine increases RV inotropy and systemic arterial pressure.
Dobutamine increases RV inotropy and lowers filling pressures. It may worsen systemic arterial hypotension unless used in combination with a vasopressor.

420
Q

VTE prophylaxis

A

** note: enox 40 mg daily NOT 1mg/kg/day OD

Aspirin may be used as prophylaxis as per harrisons

421
Q

high risk for recurrent VTE

A

take note:
minor surgery - intermediate risk
major surgery - low risk

422
Q

Stages of HF

A
423
Q

Tx for symptomatic HFref and HFpef

A
424
Q

HFpef trials

A
425
Q

Mananagement of different stages of HF

A

RAAS blockers class 1 in stage B but 2b only in HFmref and HFpef even if stage D

4 pillars are all class 1 for Class D with HFrefs

426
Q

Duration of anticoag for STEMI

A
427
Q

Risk stratification of ACS NSTEMI

A
428
Q

Max time FMC to ECG diagnosis of STEMI

A

** not from harrisons

429
Q

What is May thurner syndrome?

A

Compression of the left iliac vein by the right iliac artery

430
Q

Systolic and diastolic BP are based on ___ and ___ Korotkoff sounds

A

1st and 5th

431
Q

Very low diastolic BP (even 0) may be recorded in _____

A

chronic severe AR or
large AV fistula

because of enhanced diastolic run off

432
Q

BP is best assessed at ____ level artery

A

brachial

433
Q

Target LDL for px with familial cholesterolemia

A
434
Q

T/F Most arrhythmias do not present with palpitations

A

True

Palpitations are common among athletes, especially older endurance athletes. In addition, the enlarged ventricle of aortic regurgitation and accompanying hyperdynamic precordium frequently lead to the sensation of palpitations. Other factors that enhance the strength of myocardial contraction, including tobacco, caffeine, aminophylline, atropine, thyroxine, cocaine, and amphetamines, can cause palpitations

Most patients with palpitations do not have serious arrhythmias or underlying structural heart disease. If sufficiently troubling to the patient, occasional benign atrial or ventricular premature contractions can often be managed with beta-blocker therapy.

435
Q

Patients with psychiatric causes for palpitations more commonly report a longer duration of the sensation (>___ min) and other accompanying symptoms than do patients with other causes.

A

> 15 min

436
Q

potent peptide vasoconstrictor is released by endothelial cells. Its concentration in the plasma is elevated in patients with severe heart failure and contributes to renal vasoconstriction, sodium retention, and edema

A

Endothelin 1

437
Q

Why does edema occur in glomerulonephritis?

A

The edema that occurs during the acute phase of glomerulonephritis is characteristically associated with hematuria, proteinuria, and hypertension. In most instances, the edema results from primary retention of sodium and water by the kidneys owing to renal dysfunction. This state differs from most forms of heart failure in that it is characterized by a normal (or sometimes even increased) cardiac output.

438
Q

Drugs associated with edema

A

Mechanisms include renal vasoconstriction (nonsteroidal anti-inflammatory drugs and cyclosporine), arteriolar dilation (vasodilators), augmented renal sodium reabsorption (steroid hormones), and capillary damage.

439
Q

Difference of edema due to heart failure vs nephrotic syndrome in terms of distribution of edema

A

The distribution of edema is an important guide to its cause. Edema associated with heart failure tends to be more extensive in the legs and to be accentuated in the evening, a feature also determined largely by posture. When patients with heart failure are confined to bed, edema may be most prominent in the presacral region.

Edema resulting from hypoproteinemia, as occurs in the nephrotic syndrome, characteristically is generalized, but it is especially evident in the very soft tissues of the eyelids and face and tends to be most pronounced in the morning owing to the recumbent posture assumed during the night.

440
Q

Some murmurs may have a distinct or unusual quality, such as the “honking” sound appreciated in

A

some patients with mitral regurgitation (MR) due to mitral valve prolapse (MVP).

441
Q

early, decrescendo systolic murmur best heard at or just medial to the apical impulse

A

Acute, severe MR into a normal-sized, relatively noncompliant left atrium results in an early, decrescendo systolic murmur best heard at or just medial to the apical impulse. These characteristics reflect the progressive attenuation of the pressure gradient between the left ventricle and the left atrium during systole owing to the rapid rise in left atrial pressure caused by the sudden volume load into an unprepared, noncompliant chamber, and contrast sharply with the auscultatory features of chronic MR

442
Q

Causes of continuous murmurs

A
443
Q

Most commonly involved papillary muscle in acute severe MR from papillary muscle rupture

A

Acute, severe MR from papillary muscle rupture usually accompanies an inferior, posterior, or lateral MI and occurs 2–7 days after presentation. It often is signaled by chest pain, hypotension, and pulmonary edema, but a murmur may be absent in up to 50% of cases. The posteromedial papillary muscle is involved 6–10 times more frequently than the anterolateral papillary muscle.

The murmur is to be distinguished from that associated with post-MI ventricular septal rupture, which is accompanied by a systolic thrill at the left sternal border in nearly all patients and is holosystolic in duration

444
Q

T/F

TTE is indicated in all cases of suspected acute, severe MR

A

TTE is indicated in all cases of suspected acute, severe MR to define its mechanism and severity, delineate left ventricular size and systolic function, and provide an assessment of suitability for primary valve repai

445
Q

When is TTE indicated when a px has a grade 2 -3 midysystolic murmur?

A

TTE is indicated to evaluate a grade 2 or 3 midsystolic murmur when there are other signs of cardiac disease.

446
Q

What is the difference between the characteristic of murmur in acute severe AR vs chronic AR

A

acute, severe AR is notably shorter in duration and lower pitched than the murmur of chronic AR.

447
Q

PR in the absence of pulmonary hypertension can occur in what conditions?

A

PR in the absence of pulmonary hypertension can occur with endocarditis or a congenitally deformed valve.

448
Q

T/F
The murmur of MS is low-pitched and thus is best heard with the diaphragm of the stethoscope.

A

False

bell

bell-low

449
Q

Mid diastolic murmur during an episode of acute of rheumatic fever

A

A short, mid-diastolic murmur is rarely heard during an episode of acute rheumatic fever (Carey-Coombs murmur) and probably is due to flow through an edematous mitral valve. An opening snap is not present in the acute phase, and the murmur dissipates with resolution of the acute attack.

450
Q

A new systolic murmur in a patient with a marked fall in blood pressure after a recent MI suggests

A

myocardial rupture

451
Q

Palmar crease xanthoma is specific for

A

Type III hyperlipoproteinemia

452
Q

PMI at the epigastric area may be seen in

A

advanced COPD

453
Q

Unopposable thumb is seen in

A

Hold Oram syndrome

454
Q

Arachnodactyly and + wrist/thumb sign is seen in

A

Marfans syndrome

455
Q

Higher leg arm pressure is seen in

A

Chronic severe AR
Lower extremity PAD

systolic leg pressure usually 20 mmHg higher than systolic arm

456
Q

Higher differential BP between both arms are seen in

A

Atherosclerotic or inflammatory subclavian artery dse
supravalvular AS
Aortic coarctation
Aortic dissection

so basically all related to aorta

457
Q

Anacrotic pulse with shrill/shudder is seen in

A

AS

458
Q

triple cadence beat is seen in

A

HOCM

palpable S4 + 2 components of bisferiens pulse

459
Q

Soft s2 is seen in

A

AS (soft A2)
PS (soft P2)

460
Q

atrial REpolarization may be seen in

A

acute pericarditis
atrial infarction

461
Q

CT calcium scoring has high prognostic values among

A

Asymptomatic indivs

Agatson score
10-100 = mild
100-400 = moderat

462
Q

Which test has been found to be useful in serial documentation of aneurysm size and screening of patients at risk for abdominal aortic aneurysm?
a. Abdominal ultrasonography
b. Abdominal radiography
c. Abdominal CT scan
d. Abdominal MR

A

a. Abdominal ultrasonography

463
Q

Which of the following statements is true regarding the prognosis of renal artery stenosis due to atherosclerosis in particular?
a. It often has minor hemodynamic effects.
b. It is considered as a specifically treatable cause of secondary cause of hypertension
c. It responds well to percutaneous renal artery angioplasty
d. It is a strong predictor of morbidity- and mortality-related cardiovascular events Feedback

A

d. It is a strong predictor of morbidity- and mortality-related cardiovascular events Feedback

464
Q

Which of the following is true about therapy for ischemic heart disease?
a. Diltiazem can be combined with beta blockers in patients with normal ventricular function and no conduction disturbances.
b. Immediate release-nifedipine may be safely given in patients with elevated blood pressure during acute myocardial infarction.
c. Immediate discontinuation of a beta blocker is recommended in a patient who develops side effects from the medication.
d. Isosorbide mononitrate may be given to a patient with hypertrophic obstructive cardiomyopathy.

A

a. Diltiazem can be combined with beta blockers in patients with normal ventricular function and no conduction disturbances.

465
Q

Which of the following is TRUE of the role of salicylates and NSAIDs in the management of acute rheumatic fever (ARF)?
a. Naproxen is the drug of choice given at 20-40 mg/kg per day.
b. Salicylates or NSAIDs should be initiated in patients with chorea.
c. Recurrence of joint manifestations and fever usually recur up to 8 weeks after aspirin is discontinued.
d. They have no proven value in the treatment of carditis.

A

d. They have no proven value in the treatment of carditis.

466
Q

Which is TRUE regarding the approach to a patient with suspected tuberculous pericarditis?
a. In the presence of pericardial effusion, pericardial fluid culture is almost always diagnostic of this disease. b. Pericardiectomy should be performed in patients with thickened pericardium on biopsy before initiating anti-TB medications.
c. If with plans of surgical treatment, initiation of anti-TB medications should be delayed until post-procedure.
d. Initiation of anti-TB medications should be done if histopathologic examination reveals granulomas with caseation.

A

d. Initiation of anti-TB medications should be done if histopathologic examination reveals granulomas with caseation.

467
Q

When used in the management of venous thromboembolism, which of the following oral anticoagulants will NOT require initial therapy with unfractionated heparin or low molecular weight heparin?
a. Dabigatran
b. Edoxaban
c. Rivaroxaban
d. Warfarin

A

c. Rivaroxaban

468
Q

Which of the following is TRUE regarding the management of chronic severe MR?
a. Asymptomatic patients with severe MR in sinus rhythm and normal LV systolic function should avoid isometric exercises.
b. Apixaban may be used when AF is present in a patient with rheumatic MS and chronic severe MR.
c. Antibiotic prophylaxis for prevention of IE is indicated in all patients with chronic severe MR.
d. Vasodilators have no role in normotensive patients with isolated severe MR and normal LV systolic function

A

a. Asymptomatic patients with severe MR in sinus rhythm and normal LV systolic function should avoid isometric exercises.

469
Q

Which of the following is true regarding cholesterol absorption inhibitors?
a. It stimulates the protein NPC1L1 and blocks the intestinal absorption of cholesterol
b. It effectively inhibits the absorption of cholesterol by 80%
c. Monotherapy is indicated in patients who do not tolerate statins and in sitosterolemia
d. Monitoring of liver transaminases is not recommended when used in combination with a statin

A

c. Monotherapy is indicated in patients who do not tolerate statins and in sitosterolemia

470
Q

Which of the following statements is true regarding the management of ventricular tachycardia (VT)?
a. Medical therapy without need for an implantable cardioverter defibrillator (ICD) is adequate for patients with Brugada syndrome and recurrent syncope.
b. Chronic amiodarone therapy may be considered for individuals with ventricular fibrillation who are not candidates for ICD placement.
c. Flecainide is the drug of choice for patients with congenital long QT syndrome.
d. ICD is indicated in all patients with post-infarct ventricular arrhythmia.

A

b. Chronic amiodarone therapy may be considered for individuals with ventricular fibrillation who are not candidates for ICD placement.

471
Q

hich of the following strongly suggests ST-segment elevation myocardial infarction (STEMI)?
a. Chest pain radiating to the lower jaw accompanied by an impending sense of doom
b. New-onset arrhythmia
c. Severe chest pain lasting >30 minutes and radiating to the trapezius
d. Substernal chest pain persisting >30 minutes with diaphoresis

A

d. Substernal chest pain persisting >30 minutes with diaphoresis

472
Q

In which of the following situations is primary percutaneous coronary intervention (PCI) generally preferred over fibrinolysis in ST-segment elevation acute coronary syndrome?
a. Bleeding risk is low
b. Cardiogenic shock is present
c. Diagnosis is certain
d. Symptoms have been present for less than 2 hours

A

b. Cardiogenic shock is present

473
Q

Why is adenosine contraindicated in px with prior cardiac transplant

A

Precipitates brief AF

474
Q

T/F sustained monomorphic VT always indicates focus of automaticity/ structural abnormality

A

True

475
Q

Which of the following is the preferred intravenous parenteral medication for patients with adrenergic crisis?
a. Nitroglycerin
b. Nitroprusside
c. Nicardipine
d. Labetalol

A

b. Nitroprusside

476
Q

A 54-year-old male consulted through telemedicine. He has Hypertension, Dyslipidemia, Obesity and Erectile Dysfunction. He currently drinks three alcoholic beverages daily. His mother had a Nonfatal Myocardial Infarction (Ml) at age 55 years and he is concerned about his risk of MI. His medications are Hydrochlorothiazide, Atorvastatin and Sildenadl as needed. Which of the following components of his medical history is associated with the greatest risk for future MI?
a. Alcohol consumption
b. Dyslipidemia
c. Hypertension
d. Obesity

A

c. Hypertension

477
Q

A 70-year-old male, chronic smoker consults for dyspnea. Pertinent dndings include JVP of 10cm H20 with prominent v waves, palpable heave at the left parasternal border, hepatomegaly, ascites and lower extremity edema. Which of the following dndings is most likely TRUE of the above condition?
a. ECG: p mitrale, LV hypertrophy
b. Chest xray: normal main pulmonary artery and hilar vessels and Kerley B lines
c. 2D echo: paradoxic movement of the IV septum during systole
d. Cardiac catheterization: elevated pulmonary capillary wedge pressure and LV end diastolic pressure

A

c. 2D echo: paradoxic movement of the IV septum during systole

Hallmark of RV pressure overload

478
Q

Which of the following is TRUE of natriuretic peptides in Heart Failure?
a. Falsely elevated in obese patients
b. May also be elevated in cor pulmonale
c. Decreases with age and renal impairment
d. NT-proBNP is increased in those taking Angiotensin-receptor neprilysin inhibitor

A

b. May also be elevated in cor pulmonale

479
Q

62/M, smoker, diabetic, came for consult due to 1 month history of intermittent left calf pain which worsens after his early morning walk. He denied trauma, back pain, fever and leg weakness. VS were normal. Femoral pulses were diminished bilaterally; the left dorsalis pedis and posterior tibialis pulses were not palpable. Which is the best test to objectively assess his functional limitations?
a. Duplex ultrasonography
b. Computed Tomographic Angiography (CTA)
c. Magnetic resonance angiography
d. Treadmill testing

A

d. Treadmill testing

480
Q

60/F with hypertension and CAD complaining of 2 month history of pain, cramps and sensation of heaviness and fatigue of left lower extremity when walking in her garden, previously relieved by resting but lately noted to be associated with cold sensation and numbness of toes. She was started on intensive statin therapy, losartan and carvedilol for BP control as well as rivaroxaban and ASA. However, despite these interventions, symptoms progressed even at rest. What is the next best step?
a. 2D echo with doppler
b. Duplex ultrasonography
c. Magnetic resonance angiogram
d. Treadmill stress test

A

c. Magnetic resonance angiogram

481
Q

54/M with no vices or known comorbidities, was noted to have BP readings of 122/82, 120/84 and 126/85 in 3 separate clinic visits respectively. He has:
a. Normal blood pressure
b. Borderline hypertension
c. Hypertension stage I
d. Hypertension stage II

A

b. Borderline hypertension

482
Q

30/F G2P1 PU 32 weeks AOG previously well, came in for worsening bipedal edema not resolved with leg elevations of 3 days duration. She is otherwise asymptomatic. Initial BP was 160/110mmHg with note of proteinuria +3 on spot dipstick reading. What is the next best step?
a. Order stat pelvic ultrasound to assess fetal status
b. Restrict to very low salt diet and do home BP monitoring
c. Start methyldopa 20mg QID
d. Start IV hydralazine to target BP <140/90 mmHg

A

d. Start IV hydralazine to target BP <140/90 mmHg

Preeclampsia - Hypertension and proteinuria of ≥ 300 mg/24 hours after 20 weeks gestation. Hydralazine selectively relaxes arteriolar smooth muscle by an as-yet-unknown mechanism. Its greatest use is in the urgent control of severe hypertension or as a third-line agent for multidrug control of refractory hypertension. It is used in all trimesters of pregnancy

483
Q

Which of the following is TRUE regarding the difference between essential hypertension with high plasma renin activity (PRA) and low PRA?
a. Hypertensive patients with high PRA are more common than those with low PRA
b. Patients with high PRA may have a vasoconstriction
form of hypertension
c. Patients with high PRA may have a volume dependent hypertension
d. Patients with low PRA may have a vasocontriction form of hypertension

A

b. Patients with high PRA may have a vasoconstriction
form of hypertension

Plasma renin produces angiotensin II (Ang II) to constrict the arterioles and thereby ensure sufficient BP to deliver an appropriate rate of flow for cardiovascular homeostasis

484
Q

69/M with T2DM and hypertension, consulting for easy fatigability. He noted palpitations, chest heaviness and shortness of breath when climbing up his 3-storey house the past month. Symptoms are relieved with rest. He also noted bipedal edema in the late afternoons relieved with leg elevations. VS stable with BP 120/80 HR 84bpm, regular RR 18 cpm Afebrile with O2 at 99% On PE, there is note of a palpable precordial third heart sound. Pertinent findings on initial labs are as follows: ECG: NSSTTWCs CXR: mild cardiomegaly with no haziness or effusion Abd UTZ: mild hepatomegaly 2D echo: mildly enlarged LV with EF 59% (NV >/= 55%) ALT 42 (NV 4-36 U/L)

HbA1c 6.7% Crea 1.1 mg/dl (GFR 73) Na 136 K 4.0 CBC normal TSH normal U/A unremarkable He is maintained on perindopril 10mg OD and sitagliptin + metformin 50/1000mg BID.

According to the available clinical trials, what will be your best advise?
a. Shift perindopril to irbesartan to decrease risk of cardiovascular death or HF hospitalization based on the I-PRESERVE trial
b. Start digoxin to decrease all-cause mortality or cardiovascular hospitalization based on the DIG ancillary trial
c. Start nebivolol to decrease cardiovascular death and hospitalization based on the SENIORS trial
d. Advise exercise training to improve functional capacity and quality of life

A

d. Advise exercise training to improve functional capacity and quality of life

484
Q

30/F obese G2P2 (1012) with APAS and GDM s/p recent CS to live baby girl, preterm at 34 weeks, suddenly complained of shortness of breath with chest pain worsened with inspiration, dizziness and palpitations. BP was noted to be 70/50mmHg. Aside from hydration, what is your next best step?
a. Start IV norepinephrine and dobutamine
b. Start IV norepinephrine and give rTPA 100mg IV over 2H
c. Start dobutamine and give rTPA 100mg IV over 2H
d. Start IV vasopressin, dopamine and dobutamine

A

a. Start IV norepinephrine and dobutamine

Patient is in cardiogenic shock. For inotrope agents, when norepinephrine fails to restore perfusion, dobutamine represents the first-line agent.

485
Q

The acute decompensated heart failure (ADHF) phenotype associated with pulmonary edema is caused a. cardio-renal syndrome and hepatomegaly
b. renal insufficiency and hypoxia
c. pulmonary embolism and infection
d. new onset arrhythmia and CNS injury

A

d. new onset arrhythmia and CNS injury

486
Q

64/M with diabetes, hypertension, and chronic heart failure came in for chronic cough and angioedema for at least the past 6 months. He complains easy fatigability and shortness of breath even when brushing his teeth. He also has a 2-3 pillow orthopnea. Office BP is 150/80 which confirms his home BP monitoring which ranges at 140-150/ 85-92 mmHg. His maintenance medications include gliclazide 60mg OD, enalapril 10mg OD and HCTZ 25 mg OD. What is your next best step?
a. Add beta blocker propranolol 10mg BID to improve survival
b. Shift enalapril to irbesartan 150mg OD and observe c. Stop enalapril for a week for washout period then start beta blocker metoprolol 25mg BID before resuming enalapril 10mg OD to improve outcomes
d. Shift enalapril to telmisartan 80mg OD and add carvedilol 12.5mg BID to improve survival

A

d. Shift enalapril to telmisartan 80mg OD and add carvedilol 12.5mg BID to improve survival

Treatment of symptomatic HF includes RAAS blockers, Beta blockers and most recently SGLT2i as the pillars of pharmacotherapy. Addition of Beta blocker provides further 35% reduction in mortality.

Patients who are intolerant to ACEi due to cough or angioedema may benefit from giving ARBs as an alternative.

487
Q

60/F with long standing diabetes, hypertension and chronic heart failure NYHA II to III secondary to CAD came to the clinic with the following recent lab results: Crea 1.8 mg/dl (previously 1.2 mg/dl K 5.5 (previously 4.8) Na 136 (previously 138) +3 proteinuria on urinalysis HbA1c 7.2% (previously 7.5%) Her BP ranges 120-140/70-80. She is maintained on Valsartan 160mg OD, linagliptin 5mg OD, Insulin degludec 20 units SC OD, and furosemide 40mg prn. What is your best treatment approach?
a. Shift valsartan to hydralazine + ISDN
b. shift valsartan to spironolactone + nebivolol
c. Shift valsartan to perindopril + carvedilol
d. shift valsartan to amlodipine + metoprolol

A

a. Shift valsartan to hydralazine + ISDN

488
Q

Which of the following patients with Tachycardia-Bradycardia variant of Sick Sinus Syndrome has the greatest risk for thromboembolism?
a. 50/F with no co morbid condition
b. 60/M diabetic
c. 70/F with recent ischemic stroke
d. 80/M hypertensive

A

c. 70/F with recent ischemic stroke

CHADSVASC scores
a. 50/F with no co morbid condition -0
b. 60/M diabetic -1(DM)
c. 70/F with recent ischemic stroke 1 (age), 1(F), stroke (2) = 4
d. 80/M hypertensive 2 (age), 1 (hpn) =3

489
Q

A 25/F came for evaluation of syncope. She has no co morbid condition. Physical examination and 2d echocardiogram are within normal limits.
Her ECG showed the following: What is the next best diagnostic test to request?
a. Treadmill stress test
b. Treadmill stress echocardiogram
c. Myocardial perfusion can
d. Cardiac magnetic resonance imaging

A

d. Cardiac magnetic resonance imaging

Targeted advanced imaging—magnetic resonance imaging (MRI): amyloidosis, myocarditis, hemochromatosis, sarcoidosis, congenital heart disease (CHD), sinus of Valsalva aneurysm, aortic dissection, arrhythmogenic right ventricular cardiomyopathy

490
Q

A 55/M, hypertensive, diabetic was brought to the emergency room. Troponin is elevated. Vital signs: BP 80/50 mmHg ; HR 41 bpm ; RR 23 cpm. How should the patient be managed?
a. Give amiodarone
b. Give atropine
c. Pacemaker implantation
d. No treatment is needed at this time

A

b. Give atropine

Since with hypotension

491
Q

A 30/M had shortness of breath on exertion and peripheral edema. His vital signs are within normal limits. Physical examination revealed icteric sclerae, prominent x descent in jugular vein, with rales at bilateral bases. The cardiac impulse is displaced laterally with an audible S3. His abdomen is distended, with evidence of hepatosplenomegaly and ascites. There is bilateral peripheral pitting edema. Which among the following condition describes the case?
a. Restrictive Cardiomyopathy
b. Cardiac Tamponade
c. Dilated Cardiomyopathy
d. Brugada Syndrome

A

a. Restrictive Cardiomyopathy

Since with R sided HF symptoms

492
Q

A 25/M came in for his annual executive examination. His vital signs and physical examination were unremarkable. ECG and other laboratory tests were normal. 2D echocardiogram showed presence of multiple trabeculations in the left ventricle distal to the papillary muscles with evidence of systolic dysfunction. What should be the treatment plan for this patient?
a. Early implantable defibrillator as this patient is high risk for ventricular arrhythmias
b. Antiplatelets prevent embolic event
c. Reassurance with frequent visits
d. Fluid restriction

A

a. Early implantable defibrillator as this patient is high risk for ventricular arrhythmias

493
Q

evaluated for chest pain which started 1 month ago. Pain is at rest and is intermittent. Troponin is elevated. ECG showed STE. Coronary angiogram showed transient coronary spasm without obstruction. What is the main treatment for the patient?
a. Aspirin
b. Low molecular weight heparin
c. Calcium channel blocker
d. Statins

A
494
Q

Which among the following is a high risk feature which may entail revascularization?
a. Inability to exercise for 7 minutes under Bruce protocol
b. >0.1 mV ST segment depression after completion of stage II
c. Cessation of ST depression immediately during recovery
d. >0.2 mV ST segment depression at any stage

A

d. >0.2 mV ST segment depression at any stage

a. Inability to exercise for 7 minutes under Bruce protocol (6 minutes, so technically correct)
b. >0.1 mV ST segment depression after completion of stage II (BEFORE)
c. Cessation of ST depression immediately during recovery (AFTER 5 mins instead of immediately)
d. >0.2 mV ST segment depression at any stage

495
Q

What is the advantage of coronary computed tomographic angiography (CCTA) among other modalities?
a. Cost-effective
b. Immediate result
c. Can exclude epicardial coronary artery disease
d. Can distinguish pericardial effusion

A

c. Can exclude epicardial coronary artery disease

496
Q

A 20/F with no known co morbids presents with palpitations and occasional shortness of breath. Auscultation showed mid or late mid systolic click. Which of the following is the more likely etiology of her condition?
a. Dilated cardiomyopathy
b. Chordal rupture from myocardial infarction
c. Myxomatous degeneration
d. Rheumatic heart disease

A

c. Myxomatous degeneration

Mitral valve prolapse (MVP), also variously termed the systolic clickmurmur syndrome, Barlow’s syndrome , floppy-valve syndrome, and billowing mitral leaflet syndrome, is a relatively common but highly variable clinical syndrome resulting from diverse pathologic mechanisms affecting the mitral valve apparatus. Among these are excessive or redundant mitral leaflet tissue, which is commonly associated with myxomatous degeneration and greatly increased concentrations of certain glycosaminoglycans. MVP is the most common abnormality leading to primary mitral regurgitation (MR)

497
Q

Which of the following procedures is considered higher risk for mortality during non-cardiac surgical procedures?
A. Major thoracic surgery
B. Major abdominal surgery
C. Emergent major operation
D. Carotid endarterectomy

A

C. Emergent major operation

498
Q

A 23/F (G3P1) came in for exertional dyspnea which started 2 weeks. She delivered a full-term infant 1 month ago thru stat caesarean section because of preeclampsia. Echocardiography demonstrates a dilated left ventricle size with global hypokinesia. Which of the following risk factors in the patient predisposed her to her present condition?
A. Obesity
B. Preeclampsia
C. Young maternal age
D. Multiple miscarriage

A

B. Preeclampsia

Risk factors: - Increased maternal age - Increased parity - Twin pregnancy - Malnutrition - Use of tocolytic therapy for premature labor, and - Preeclampsia or toxemia of pregnancy

499
Q

A 42/M presents at the emergency room with on and off episodes of chest pain. The pain occurs at rest and is relieved with sublingual nitrogylcerin. Cardiac physical examination is unremarkable. There was a transient ST segment elevation noted in the ECG tracings. What other diagnostic hallmark can be observed in this case?
A. Coronary vasospasm during angiography
B. Elevated C-reactive protein (CRP levels)
C. Segmental wall motion abnormalities in 2d echocardiogram
D. Hypercholesterolemia

A

A. Coronary vasospasm during angiography

Prinzmetal’s Variant Angina
This is described as a syndrome of severe ischemic pain that usually occurs at REST and is associated with STsegment elevation. Prinzmetal’s variant angina (PVA) is caused by focal spasm of an epicardial coronary artery with resultant transmural ischemia and abnormalities in left ventricular function that may lead to acute MI, ventricular tachycardia or fibrillation, and sudden cardiac death.

The cause of the spasm is not well defined, but it may be related to hypercontractility of coronary arterial smooth muscle due to adrenergic vasoconstrictors, leukotrienes, or serotonin.

Coronary angiography demonstrates transient coronary spasm as the diagnostic hallmark of PVA. Atherosclerotic plaques in at least one proximal coronary artery occur in about half of patients.

Hyperventilation and intracoronary acetylcholine have been used to provoke focal coronary stenosis on angiography or to provoke rest angina with ST-segment elevation to establish the diagnosis.

500
Q

A 36/F, obese II, hypertensive and diabetic came to your clinic complaining of on and off chest pain. She has unremarkable cardiac physical examination. Which of the following will be the best choice to rule out high risk feature of ischemia?
A. 12-L ECG
B. Chest Xray
C. Treadmill Stress Test
D. 24-hour holter monitoring

A

C. Treadmill Stress Test

501
Q

Which of the following findings in the cardiac nuclear scan is high risk for ischemia?
A. Three ischemic zones seen in the myocardium imaging.
B. Change in left ventricular ejection fraction during exercise
C. Positive ECG changes in the inferior wall
D. 0.1 mV ST segment depression in leads II and III

A

B. Change in left ventricular ejection fraction during exercise

502
Q

In which of the following case would be most appropriate to use coronary CT angiography?
A. 60/M with complete right bundle branch block
B. 70/F amputee, hypertensive and diabetic
C. 72/M with no known comorbidities and does walking everyday for exercise
D. 51/M with ongoing chest pain

A

C. 72/M with no known comorbidities and does walking everyday for exercise

CT calcium scoring –> high cardiac prognostic value in ASYMPTOMATIC px

503
Q

A 50/M came to the clinic with a referral from a dentist asking for possible antibiotic prophylaxis for his heart condition prior to tooth extraction. He has mitral valve prolapse with moderate regurgitation with no history of endocarditis. He is maintained on Losartan 50 mg once a day along with Diabetes Mellitus type 2 medications (Insulin Glargine, Metformin and Glipizide). Which of the statements is appropriate?
A. Give Amoxicillin 500 mg single dose prior to dental extraction
B. Give Cefixime 200 mg single dose prior to dental extraction
C. Give Cephalexin 500 mg single dose prior to dental extraction
D. No prophylaxis is needed

A

Answer: D. No prophylaxis is needed

504
Q

A 3A 78/M known hypertensive, consulted for progressive shortness of breath upon exertion. Physical examination findings: BP 110/70 mmHg, HR 95 bpm, RR 18 cpm. JVP = 3 cm H20. Clear breath sounds. Adynamic precordium, Apex beat 6th ICS LMCL, distinct S1, silent S2, Grade 3/6 Systolic ejection murmur with late peaking over the 2nd intercostal space, right parasternal border with radiation to the carotids. The carotid impulse rises slowly with late peaking. What is the most likely etiology of the patient’s condition?
A. Calcific
B. Congenital
C. Hypertensive Heart Disease
D. Rheumatic

A

A. Calcific

505
Q

65/M market vendor, with BMI 31, diabetic, smoker, had 2 episodes of myocardial infarction, consulted due to 2-month history of feeling of “fatigue” in his legs every time he had long walks, sometimes associated with muscle cramps. He also reported hair loss on the legs. He denies any trauma to the legs. Which of the following diagnostic tests will help you objectively assess the functional limitation of this patient?
A. ABI measurement
B. Treadmill Testing
C. Duplex Ultrasonography of the lower extremities
D. Computed Tomographic Angiography (CTA)

A

B. Treadmill Testing

to assess functional limitations objectively

Since with claudication, may be advised to exercise regularly

505
Q

53/F public school teacher consulted due to 8-month history of worsening swelling of the legs, not associated with pain. She has hypertension and has good functional capacity. She mentioned that she usually noticed leg swelling after teaching in class and improves upon waking up in the morning. What is the next best step to diagnose the condition of this patient? A. Review patient’s medication
B. Do conventional venography
C. Request for venous duplex ultrasound
D. Request for 2D Echocardiography

A

C. Request for venous duplex ultrasound

506
Q

68/F hypertensive and diabetic consulted due to bilateral leg swelling and “ulcer-like” lesion near the left ankle. Which of the following findings is associated with venous insufficiency?
A. Nonpitting edema
B. Hypopigmentation
C. Hypertrophic skin
D. Eczema

A

D. Eczema

507
Q

38/F with no known co-morbids came in at the ER due to sudden onset severe abdominal pain and unexplained breathlessness. She takes OCP and is a tennis player. She underwent surgery for PCL tear repair 1 month ago but had decreased mobility since after surgery. At the ER, VS are: BP 80/50 mmHg, HR 125 bpm, RR 26cpm, Temp 36.7C, O2 sats 82% at room air, improved to 85% at 5LPM. Lungs are clear

What diagnostic examination will you do to confirm your assessment?
A. D Dimer
B. Echocardiography
C. Pulmonary angiography
D. Chest CT scan with IV contrast

A

D. Chest CT scan with IV contrast

508
Q

79/M stroke patient and bedridden for 1 year was sent to ER due to sudden onset difficulty breathing. The home care giver reported that oxygen saturation is 80%. At the ER, VS are as follows: BP 70/40, HR 132, RR 30, Temp 36.6C, O2 sats 83% at 10LPM. A bedside 2D Echo was done showing: RV enlargement, hypokinetic RV free wall with hyperkinetic RV apex and interventricular septal flattening. What is the best management for this patient?
A. Fibrinolytic therapy
B. Embolectomy
C. IVC filter
D. Anticoagulation

A

B. Embolectomy

509
Q

.35/F, gave birth 2 weeks ago, was referred to you due to new onset exertional dyspnea and increasing bipedal edema. Patient also verbalized that she would be awakened multiple times at night because of shortness of breath described as “feeling of suffocation”, which is relieved after sitting upright. Which of the following statements is consistent with the pathophysiology of her symptom?
A. Increased bronchial arterial pressure
B. Increased sensitivity of the respiratory center to arterial PCO2
C. Redistribution of fluid from splanchnic circulation
D. Decreased blood flow to respiratory muscles

A

A. Increased bronchial arterial pressure

Paroxysmal nocturnal dyspnea (PND) refers to episodes of shortness of breath that awaken a patient suddenly from sleep with feelings of anxiety and suffocation and require sitting upright for relief. In contrast to orthopnea, PND usually occurs after prolonged recumbency, is less predictable in occurrence, and may require 30 min or longer in the upright position for relief. Episodes are often accompanied by coughing and wheezing (so-called cardiac asthma) thought to be due to increased bronchial arterial pressure leading to airway compression and interstitial pulmonary edema causing increased airway resistance.

510
Q

76/F, with BMI 30, history of hypertension, diabetes and 3-vessel CAD consulted due to bipedal edema, bloatedness, early satiety, cough and shortness of breath even on light activities like changing clothes. Which of the following counterregulatory hormones is upregulated in her condition?
A. Renin
B. Neprilysin
C. Endothelin
D. Natriuretic peptide

A

D. Natriuretic peptide

While RAAS and SNS activation contributes to disease progression in HF, a number of counterregulatory hormones are upregulated and exert beneficial effects on the heart, kidney, and vasculature. These include the natriuretic peptides (atrial natriuretic peptide [ANP] and B-type natriuretic peptide [BNP]), prostaglandins (prostaglandin E1 [PGE1] and prostacyclin [PGI2]), bradykinin, adrenomedullin, and nitric oxide.

511
Q

A 55-year-old male with a history of hypertension and a recent ischemic stroke is eligible for thrombolytic therapy. His blood pressure is 200/110 mmHg. What should be the target blood pressure before initiating thrombolysis?
A. Less than 185/110 mmHg
B. Less than 195/110 mmHg
C. Less than 190/100 mmHg
D. Less than 185/100 mmHg

A

A. Less than 185/110 mmHg

511
Q

Which of the following diagnostic tests is highly accurate and is considered as a quantitative tool for evaluation of left ventricular mass, volumes, and function?
A. 2D Echocardiography
B. Cardiac MRI
C. Coronary angiography
D. Transesophageal Echocardiography

A

B. Cardiac MRI

Cardiac magnetic resonance imaging (CMR) has emerged as a highly accurate and quantitative tool for evaluation of left ventricular mass, volumes, and function and for determining specific causes of HF (e.g., ischemic cardiomyopathy, myocarditis, amyloidosis, hemochromatosis).

512
Q

What is the proper sequence of the 5 Cs for out-of-hospital cardiac arrest during a pandemic?
A. Call → Cover → Check → Connect → Compress
B. Check → Call → Cover → Compress → Connect
C. Call → Cover → Check → Compress → Connect
D. Check → Call → Cover → Connect → Compress

A

B. Check → Call → Cover → Compress → Connect

In the Philippines, The Philippine Heart Association has clear guidelines on how to perform safe Lay Rescuer CPR in times of COVID for Out-of-Hospital Cardiac Arrests.
These guidelines are called 5 C’s for Lay CPR amidst COVID:
1. Check – on the victim from a safe distance
2. Call – quickly call for help
3. Cover – protect both rescuer with a proper face mask and victim with a face mask or cloth.
4. Compress – do hands-only CPR
5. Connect – go for early defibrillation with an AED

513
Q

Mr. J is a 30 year old obese man who consulted you due to exertional dyspnea which started about 12 weeks ago after a bout of viral upper respiratory tract infection. 2D Echocardiogram showed a dilated left ventricle with global hypokinesia EF of 32%. BP is 130/80. Which of the following is an appropriately dosed choice of a CORNERSTONE medication to start on him?
a.Amlodipine 5mg once a day
b.Carvedilol 25mg three times a day
c.Enalapril 5mg once a day
d.Valsartan 320mg twice a day

A

c.Enalapril 5mg once a day

Wrong dose valsartan: 160 BID max only

514
Q

A 70 year old man is referred to you for exertional dyspnea and orthopnea. He hypertensive, non-diabetic, and does not smoke. Pro-BNP is 4,000 pg/mL, and ECG shows sinus rhythm with left ventricular hypertrophy. 2D Echocardiogram shows Ejection fraction of 55% with adequate wall motion and contractility. Coronary Angiogram had no significant findings. Which among the following trials showed a significant reduction in mortality for this specific population?
a. CHARM-Preserve Trial
b. DIG Trial
c. PARADIGM-HF Trial
d. None of the Above

A

d. None of the Above

PARADIGM HF is for HFref

515
Q

A 37 year old man, visibly anxious, consults to your clinic for a check up. His blood pressure is 143/91. He asks you if he is hypertensive and if he should start medication. He never checked his blood pressure at home before. Which of the following statements is accurate? (Based on Philippine Hypertension CPG 2020).
a. He cannot be classified as hypertensive yet, and a 24 hour ambulatory blood pressure monitoring or home blood pressure measurements can provide further data.
b. He has white coat hypertension, for which there is no specific treatment.
c. He is hypertensive and should be started on a drug such as Losartan 50mg once a day.
d. He is hypertensive, an elevated blood pressure above 140/90 on the next visit should prompt pharmacological therapy

A

a. He cannot be classified as hypertensive yet, and a 24 hour ambulatory blood pressure monitoring or home blood pressure measurements can provide further data.

516
Q

A 30 year old blogger who takes oral contraceptive pills presented to the ER for right leg swelling after returning from the United States to the Philippines via plane. While at the ER, she develops sudden tachycardia 120bpm and dyspnea, saturations decreased to 88%. Blood pressure is 120/70 mmHg. D-Dimer was normal and 2DEchocardiogram showed no right ventricular dysfunction. Stat CT scan of the pulmonary artery showed segmental filling defects. Which of the following is the correct assessment of her condition?
a. Panic attack as the D-dimer is low
b. Pulmonary embolism, low risk
c. Pulmonary embolism, submassive
d. Pulmonary embolism, massive

A

b. Pulmonary embolism, low risk

Since normal BP and 2d echo

517
Q

In which of the following patients is ultrasonography for screening of abdominal aortic aneurysm recommended?
a. A 48 year old woman with a popliteal artery aneurysm
b. A 60 year old man, smokes 1 pack cigarette per day, with a brother who has an abdominal aortic aneurysm c. A 70 year old man who stopped smoking at the age of 30 years old
d. A or C

A

d. A or C

Abdominal ultrasound is useful for serial documentation of aneurysm size and can be used to screen patients at risk for developing an aortic aneurysm.

Screening by ultrasonography is recommended for men aged 65–75 years who have ever smoked. In addition, male and female siblings or offspring of persons with abdominal aortic aneurysms, as well as individuals with thoracic aortic or peripheral arterial aneurysms, should be considered for screening for abdominal aortic aneurysms.

518
Q

A 38 year old man presented to the emergency room for severe chest pain described as tearing and the worst in his life, radiating from the chest to the neck then the back. CT scan showed an intimal tear involving the ascending aorta until the suprarenal aorta. BP is 80/50 mmHg. Which of the following is the best management?
a. Control of Blood pressure and heart rate, conservatize
b. Delay surgery until stabilized
c. Immediate coronary angioplasty
d. Immediate surgical referral

A

d. Immediate surgical referral

519
Q

Which of the following scenarios is most consistent with a patient experiencing chronic venous insufficiency?
a. Leg is pale and cool, improves with standing
b. Legs are heavy and bilaterally swollen after a long day of walking, and recede when elevated at night
c. Legs have missing appendages such as hair
d. Leg is unilaterally swollen, erythematous, tender, and with visible superficial veins

A

b. Legs are heavy and bilaterally swollen after a long day of walking, and recede when elevated at night

520
Q

A 50 year old man presents to a provincial hospital for chest discomfort. 12L ECG was done showing acute inferior wall ST elevation myocardial infarction. The hospital is non-PCI capable. There is a nearby private hospital with a cathlab and an available interventional cardiologist. A decision is made to transfer the patient for primary PCI. How long is the target time from Door In to provincial hospital to Door Out of provincial hospital (DIDO)?
a. 15 minutes
b. 30 minutes
c. 90 minutes
d. 120 minutes

A

b. 30 minutes

521
Q

Which of the following patients has a class I indication for open heart surgery
a.80M Symptomatic severe aortic regurgitation with poor life expectancy less than 6 months
b.55M Asymptomatic severe aortic regurgitation with LV ejection fraction 40 % LV end diastolic dimension 61mm
c.60F Asymptomatic severe aortic regurgitation with LV ejection fraction 53% LV end diastolic dimension 55mm
d.None of the above

A

b.55M Asymptomatic severe aortic regurgitation with LV ejection fraction 40 % LV end diastolic dimension 61mm

522
Q

Which of the following statements is true about ultrafiltration in acute heart failure as seen in the CARESS-HF trial?
a. Ultrafiltration reduced heart failure hospitalizations b. Ultrafiltration afforded more weight loss (hence more water clearance) than stepped pharmacological care
c. Ultrafiltration group showed worse renal outcomes and creatinine than stepped pharmacological care
d. Ultrafiltration group had less adverse events such as bleeding complications

A

c. Ultrafiltration group showed worse renal outcomes and creatinine than stepped pharmacological care

In the Cardiorenal Rescue Study in Acute Decompensated Heart Failure (CARRESS-HF) trial, 188 patients with ADHF and worsening renal failure were randomized to stepped pharmacologic care or UF. The primary endpoint was a change in serum creatinine and change in weight (reflecting fluid removal) at 96 h. Although similar weight loss occurred in both groups (~5.5 kg), there was a rise in serum creatinine among patients allocated to the UF group. Deaths and hospitalizations for HF were no different between groups, but there were more adverse events in the UF group, mainly due to kidney failure, bleeding complications, and intravenous catheter-related complications.

523
Q

A 60 year old lady, recently diagnosed with breast cancer, presented to the emergency room for leg pain. It started 4 days ago after she slipped in the bathroom. Since then she was confined to the bed unable to move due to pain. There was no numbness or weakness. On examination, there was swelling of the entire right leg, with pitting edema and collateral superficial veins, no tenderness. The right leg circumference is 3cm larger than the left.
What is her well’s score for DVT?
a. 4
b. 5
c. 6
d. 7

A

d. 7

Active cancer: +1
Bedridden for >3 days: +1
Immobilization: +1
Entire leg swollen: +1
Calf swelling ≥3 cm: +1
Pitting edema: +1
Collateral superficial veins: +1

524
Q

A 45-year-old man, chronic alcohol binge drinker, with progressive shortness of breath has the following findings on 2D echo: massive global dilation and thinning of the LV wall, with enlarged LA. What is the underlying mechanism of the patient’s cardiac conditions?
a. Direct myocardial toxicity of acetaldehyde
b. Decreased angiogenic reserve with inhibition of secretion of VEGF
c. Depression of cardiac function from a systemic inflammatory response
d. Intense sympathetic activation of the myocardial autonomic innervation

A

a. Direct myocardial toxicity of acetaldehyde

525
Q

Which of the following findings during exercise stress electrocardiography of a patient suggests a severe Ischemic Heart Disease and a high risk of future adverse events?
a. Appearance of ventricular arrhythmias
b. Failure to achieve 85% of the maximal predicted heart rate for age and sex
c. ST segment depression persisting beyond 5 minutes after termination of the exercise
d. Upsloping of ST-segment before completion of stage II of the Bruce protocol

A

c. ST segment depression persisting beyond 5 minutes after termination of the exercise

526
Q

When used in the management of venous thromboembolism, which of the following oral anticoagulants will NOT require initial therapy with unfractionated heparin or low molecular weight heparin?
a. Dabigatran
b. Edoxaban
c. Rivaroxaban
d. Warfarin

A

c. Rivaroxaban

527
Q

Which is a correct monitoring diagnostic protocol for acute rheumatic fever?
a.Inflammatory markers should be repeated every one to two weeks until normalization
b.2D Echocardiography should be repeated after one week
c.Throat swab culture for Group A streptococcus should be repeated after antibiotics
d.If initial ECG shows PR prolongation, 12-lead ECG should be repeated weekly

A

a.Inflammatory markers should be repeated every one to two weeks until normalization

528
Q

A 38-year-old female with chronic kidney disease stage 4 from chronic glomerulonephritis, came to the emergency room for a 2-day history of severe left-sided chest pain radiating to her left shoulder, worse at night when lying down. She has a known allergy to mefenamic acid, which resulted in an ER admission in the past due to bronchospasm. Pertinent physical examination findings include: stable vital signs, no fever, distinct S1 and S2, and presence of pericardial friction rub on auscultation. Electrocardiogram revealed diffuse ST segment elevations in leads I, II, aVF, V3-V6. Echocardiography revealed thickened pericardium with no pericardial effusion. Additional work-up revealed negative troponins, autoimmune, and viral markers. What is the most appropriate medical therapy for this patient?
a. Anticoagulation with enoxaparin
b. Colchicine 0.5 mg twice daily for 3 months
c. Indomethacin 50 mg thrice daily for 2 weeks
d. Prednisone 1 mg/kg/day for 4 days then tapered

A

d. Prednisone 1 mg/kg/day for 4 days then tapered

529
Q

Which of the following statements is true regarding chronic venous insufficiency?
a. It is usually caused by disease of the superficial venous system.
b. Primary deep venous insufficiency is caused by valvular incompetence from previous deep vein thrombosis.
c. AV fistulas can cause secondary deep venous insufficiency.
d. Most veins remain occluded after an episode of deep vein thrombosis.

A

c. AV fistulas can cause secondary deep venous insufficiency.

529
Q

A 65/M with renal artery stenosis and bronchial asthma, on treatment with LABA + ICS inhaler, was admitted for dyspnea. Chest x-ray on admission showed pulmonary congestion. He was then started on furosemide infusion. He developed sudden onset chest pain on his 3rd hospital day. Tests showed ST depression on V5-V6 on ECG and elevated serial Troponin I, prompting management for acute non-ST elevation myocardial infarction. Monitoring after 2 days showed BP 140-150/80-90, HR 105-110. Which of the following medications are appropriate to manage his blood pressure?

a.Captopril 25 mg BID
b.Propranolol 40 mg TID
c.Terazosin 2 mg OD
d.Verapamil 120 mg BID

A

d.Verapamil 120 mg BID

a.Captopril 25 mg BID - cannot be used in bilateral renal artery stenosis
b.Propranolol 40 mg TID- px has asthma
c.Terazosin 2 mg OD - not first line

530
Q

Massive pulmonary embolism can produce severe substernal pain that may mimic an acute myocardial infarction (MI). This is attributed to which of the following mechanisms?
A. Distention of the pulmonary artery
B. Increase in intrathoracic pressures
C. Involvement of adjacent pleural surface
D. Left ventricular wall stress

A

A. Distention of the pulmonary artery

531
Q

A 32-year-old man is evaluated during an initial office visit. He has no symptoms and no significant medical history. He takes no medications. On PE, BP is 120/70 mmHg, HR 64/min. Cardiac examination reveals a grade 1/6 decrescendo early diastolic murmur heard best at the left lower sternal border. Femoral pulses are equal. Which of the following is the most likely cause of the patient’s murmur?
A. Aortic coarctation
B. Atrial septal defect
C. Bicuspid aortic valve
D. Mitral stenosis

A

C. Bicuspid aortic valve

532
Q

How should HF with IDA be tx?

A

IV iron

No benefit: EPO analogues, oral iron

532
Q

Which of the following is TRUE about palpitations?
A. Anxiety states are the most common cause of palpitations.
B. Data suggest that Holter monitoring is of vast clinical utility in patients with recurrent, unexplained palpitations.
C. Most arrhythmias are associated with palpitations.
D. Palpitations are common among older endurance athletes.

A

D. Palpitations are common among older endurance athletes.

Palpitations are brought about by cardiac (43%), psychiatric (31%), miscellaneous (10%), and unknown (16%) causes, according to one large series.
* Most arrhythmias are not associated with palpitations. * Data suggest that Holter monitoring is of limited clinical utility, while the implantable loop recorder and mobile cardiac outpatient telemetry are safe and possibly more cost-effective in the assessment of patients with (infrequent) recurrent, unexplained palpitations.
* Palpitations are common among athletes, especially older endurance athletes.

533
Q

Indication for endomyocardial biopsy in px with myocarditis

A

New onset HF + conduction or ventricular tachyarrhythmias suggestive of sarcoidosis or giant cell myocarditis

Tx for sarcoidosis = high dose glucocorticoids +/- methotrexate

Tx for giant cell myocarditis –> glucocorticoids

534
Q

When is fever and PMN leukocytosis expected to occur in STEMI

A

Fever 38 deg - 1st week

PMN leukocytosis appears within few hrs and can last for 3-7 days. WBC often reaches 12-15k

535
Q

How do you differentiate Type 4a vs 4b vs 4c

A

Type 4a = acute MI within 48 hrs after index procedure
Type 4b= stent thrombosis >48h
Type 4c = stent REstenosis

Type 4 = 5x elevation in troponin
Type 5 = 10x elev

536
Q

When is PCI preferred vs fibrinolysis

A

Doubtful diagnosis
cardiogenic shock
inc bleeding risk
ssx >= 2-3h

537
Q

Indication for full anticoagulation in STEMI px (LMWH in px then at least 3 mos of warfarin)

A

Anterior infarct
Severe LV dysfunction
AF
HF
inc risk of VTE
Mural thrombus
Hx of embolismf

538
Q

Infarct of >= ___ % will lead to cardiogenic shock

A

40%

20-25% = hemodynamic evidence of abnormal LV func

539
Q

Which of the following conditions can lead to heart failure with either reduced or preserved ejection fraction?
A. Aging
B. Chronic anemia
C. Infiltrative disorders
D. Regurgitant valvular disease

A

C. Infiltrative disorders

540
Q

Which of the following neurohormonal mechanisms contributes to short-term adaptation to heart failure but eventually results in endorgan changes increasing morbidity and mortality?
A. Decreased efferent sympathetic tone
B. Decreased permeability of renal collecting ducts
C. Decreased vasoconstriction of peripheral vasculature D. Increased AVP secretion from the pituitary gland

A

D. Increased AVP secretion from the pituitary gland

The rest should also be increased

541
Q

A 51-year-old woman comes to the ER for progressive worsening dyspnea over the past 4 hours. She has hypertension, and is on carvedilol, amlodipine, and enalapril with a usual home BP of 120/80 mmHg. She arrived last night from a weekend trip and forgot to take her medications. BP is 170/100 mmHg, HR 110/min, RR 28/min, SpO2 97% on room air. On physical exam, there is no jugular venous distention, with minimal bibasal crackles and an S3 heart sound on chest auscultation. Labs are pending. Which of the following suggested drugs will you give your patient?
A. Diuretics
B. Inotropic therapy
C. Opiates
D. Vasodilators

A

B. Inotropic therapy

542
Q

Which of the following has been demonstrated to improve clinical outcomes among patients with heart failure with reduced ejection fraction (HFrEF)?
A. Amlodipine
B. Diltiazem
C. Fish oil
D. Rosuvastatin

A

C. Fish oil

543
Q

Which of the following physical findings is consistent with aortic regurgitation?
A. Increase in murmur intensity with inspiration
B. Narrow pulse pressure
C. Radial-to-femoral pulse delay
D. Tachycardia

A

D. Tachycardia

544
Q

Which of the following is TRUE about the auscultatory findings in patients with mitral valve prolapse?
A. A frequent finding is a mid- or late-systolic ejection click.
B. Squatting and isometric exercises increase the intensity of the clickmurmur complex.
C. Systolic clicks may be multiple and may be followed by a mid-late systolic crescendo-decrescendo murmur. D. The murmur will radiate to the axilla and back with posterior leaflet prolapse.

A

C. Systolic clicks may be multiple and may be followed by a mid-late systolic crescendo-decrescendo murmur.

545
Q

A 56-year-old presents to the ER with chest pain of 20 minutes duration. He is subsequently diagnosed with anterior wall STEMI and received thrombolytic therapy with tPA. Soon after administration of tPA, the ECG changes revert to baseline. He is monitored on telemetry for 48 hours without any arrhythmias. He is able to walk around the ward without difficulty. Further diagnostic tests during the immediate discharge period should include which of the following?
A. Heart rate-limited exercise treadmill test, 1 week post-MI
B. Nuclear imaging with pharmacologic stress C. Stress echocardiogram
D. Symptom-limited exercise treadmill test, 1 week post-MI

A

A. Heart rate-limited exercise treadmill test, 1 week post-MI

D is done 4-6 weeks after

546
Q

A 55-year-old man with hypertension and ischemic heart disease consults you for erectile dysfunction. He is on diltiazem, metoprolol and combination losartan/ hydrochlorothiazide. Which drug is the most likely culprit of his current concern?
A. Diltiazem
B. Hydrochlorothiazide
C. Losartan
D. Metoprolol

A

D. Metoprolol

547
Q

A 56-year-old man comes to the emergency room for chest discomfort. He reports working on his car 2 hours ago when he felt some heaviness over his left chest. He says that this has not happened before. Medical history is pertinent for hypertension controlled on enalapril. He is a non-smoker. On PE, BP 120/70 mmHg, HR 77/min. Rest of systemic PE is unremarkable. A baseline ECG is non-diagnostic, and biomarkers on admission are not elevated. You decide to observe him in the ER. After 12 hours, he asks if he can go home as the chest discomfort did not recur. A repeat troponin level remains low. What is the next step in the management of this patient?
A. Observe the patient for a total of 24 hours with serial troponin measurements.
B. Do a stress study to provoke ischemia.
C. Admit the patient for coronary angiogram.
D. Discharge the patient.

A

B. Do a stress study to provoke ischemia.

548
Q

A 45-year-old man comes to the emergency room due to chest pain. He has hypertension and is on losartan and carvedilol. He presents with a 4-hour history of chest pain associated with dyspnea and diaphoresis. On arrival, he is hypotensive and has bibasal crackles. He is started on dobutamine and is given morphine sulfate for the pain. ECG showed ST-segment depression on leads V5-V6. His troponins are elevated. What is the best management option for this patient? A. Early invasive strategy
B. Delayed invasive strategy
C. Immediate invasive strategy
D. Trial of optimal medical management prior to invasive strategy

A

C. Immediate invasive strategy

549
Q

Among patients with suspected or confirmed aortic dissection, which of the following drugs can be used to lower systolic BP if sodium nitroprusside is unavailable?
A. IV verapamil
B. IV hydralazine
C. Oral metoprolol
D. Sublingual clonidine

A

A. IV verapamil

550
Q

Which of the following is the most appropriate management at this time?
A. Obtain cardiac magnetic resonance imaging
B. Perform coronary angiography
C. Proceed with mitral valve repair
D. No further management is required; she may proceed with pregnancy without interventions

A

C. Proceed with mitral valve repair

551
Q

A 58-year-old is referred to you for preoperative risk assessment prior to colorectal surgery. He is recently diagnosed with colorectal adenocarcinoma. He had a myocardial infarction 3 years prior and underwent revascularization with a drug-eluting stent. He is on lowdose aspirin, rosuvastatin, lisinopril and carvedilol. He has no other comorbid illnesses and no other medications. He reports being able to walk 4 blocks to work, and is able to exercise most days of the week by jogging up a hill.

What is the next step in this patient’s management?
A. Perform coronary angiography with intent to revascularize if needed
B. Perform pharmacologic stress testing
C. Perform a resting echocardiogram
D. No further testing is needed, patient may proceed with surgery

A

D. No further testing is needed, patient may proceed with surgery

552
Q

Most common valve lesion among adults with VHD

A

symptomatic valvular AS

most common cause: degenerative calcification

553
Q

Most common cause of death in aortic stenosis

A

CHF

554
Q

Treatment of choice for acute severeAR

A

Urgent surgery usually within 24h

short lived: IV diuretics and vasodilators

if chronic usually surgery is delayed >1yr from onset of ssx

555
Q

Optimal time of surgery for severe AR

A

After onset of LV dysfunction but prior to development of severe ssx

556
Q

Percutaneous commisurotomy is the procedure of choice in

A

<45 y/o + pliable valve
pregnant with MS + pulmonary congestion

successful if > 50% decrease in MV gradient and 2x ofMV area

557
Q

Which of the following statements is true about physical examination findings in aortic stenosis:
a. Atrial fibrillation often occurs early in the disease course, and may signal mitral valve involvement
b. Pulse pressure widening and systolic blood pressure decline may happen in later stages of aortic stenosis
c. There is often an ejection, mid-systolic murmur that begins shortly after S1
d. The carotid artery rises sharply, thus the classic pulsus parvus et tardus

A

c. There is often an ejection, mid-systolic murmur that begins shortly after S1

pulsus parvus et tardus is weak and late pulse

558
Q

A 54 year old woman complained of progressive fatigue and dyspnea. A year ago, she also complained of mouth and eye drying, and difficulty swallowing and swallowing. She denies fever, weight loss and other constitutional symptoms. On physical examination, she is awake, and not in distress. She has limited mouth opening and cutaneous thickening at the dorsal aspects of the hands. Vital signs are normal, there’s a holosystolic murmur at the 5th ICS left parasternal border. 2D echo which showed markedly increased PAP (45 mmHg). Under what group classification of pulmonary hypertension does the patient belong? (based on the WHO Classification)
a. Group I
b. Group II
c. Group III
d. Group IV

A

a. Group I

PAH with connective tissue dse = Group 1

559
Q

Which of the following is not a basic laboratory test recommended in the initial evaluation of hypertension? a. Hba1c
b. Serum potassium
c. Serum triglyceride
d. Urinary albumin excretion

A

a. Hba1c

FBS dapat

560
Q

A 58/M, has been complaining of intermittent claudication for three months now. He is also hypertensive on Losartan 50 mg/tab but is not compliant with his medications. He is a 30-pack year smoker. Which of the following is NOT true regarding his condition?
a. This condition affects mostly the proximal vessels
b. The internal elastic lamina is preserved
c. Migratory superficial vein thrombophlebitis is seen
d. Later stages are characterized by perivascular fibrosis

A

a. This condition affects mostly the proximal vessels

561
Q

What is the most common cause of midsystolic murmur among adults?
A. Dilation of ascending aorta
B. Pulmonary artery stenosis
C. Aortic stenosis
D. Aortic regurgitation

A

C. Aortic stenosis

562
Q

A 60-year-old man presents with acrocyanosis. Which of the following is consistent with the above nding?
A. It indicates signicant right-to-left shunting
B. It indicates right-to-left shunting at the great vessel in the presence of secondary pulmonary hypertension
C. It may be aggravated by beta-blockers
D. Consider possible advanced right-sided heart failure

A

C. It may be aggravated by beta-blockers

Acrocyanosis (peripheral cyanosis) indicates reduced extremity blood flow due to small vessel constriction

563
Q

Which of the following PE ndings indicate an increased venous volume?
A. JVP of 8 cmsH2O with patient lying supine at 30degree angle
B. Abdominal bruit on ausculations
C. Presence of Kussmaul’s sign
D. Presence of venous pulsations above the clavicle while sitting

A

D. Presence of venous pulsations above the clavicle while sitting

Use of the sternal angle as a reference point leads to systematic underestimation of CVP
Kussmaul’s sign is dened by either a rise or a lack of fall of the JVP with inspiration – Classically associated with constrictive pericarditis – Also seen in restrictive cardiomyopathy, massive TPE, RV MI, advanced LV failure, and as isolated finding in patients after cardiac surgery without other hemodynamic abnormalities

564
Q

In which scenario would a head-up tilt (HUT) test be most useful?
A. Work-up of a patient with arrythmia and severe CAD B. When suspecting exaggerated vagal tone as a cause of syncope
C. Document presence of severe aortic stenosis
D. When considering arrhythmia as a cause of a patient’s severe cerebrovascular disease

A

B. When suspecting exaggerated vagal tone as a cause of syncope

Exaggerated activation of a central reflex in response to HUT: ↑ HR ↓ BP ↓ HR (indicates neurally mediated hypotension) In patients with structural heart disease,
HUT may be indicated in those with syncope, in whom other causes (e.g., asystole, ventricular tachyarrhythmias) have been excluded

565
Q

Among these patients with atrial brillation of less than 48 hours duration, who will you recommend cardioversion without an initial anticoagulation?
A. 65-year-old woman with non-rheumatic MS
B. 40-year-old woman with rheumatic MS
C. 25-year-old man with hypertrophic cardiomyopathy and marked left atrial enlargement
D. 60-year-old man with one episode of embolic stroke 6 months earlier

A

A. 65-year-old woman with non-rheumatic MS

AF<48 hrs: cardioversion in the absence of anticoagulation may be done IF low risk for TE:
§ CHA2DS2-VASc of 0 to 1
§ NO prior embolic episodes
§ NOT rheumatic MS
§ NOT hypertrophic CMP with marked LAE

566
Q

What is the utility of treadmill exercise testing in patients with advanced heart failure?
A. Assess maximum tolerable activities
B. Assess minimum exercise that induces dyspnea
C. Assess need for cardiac transplantation
D. Assess risk of developing potentially fatal arrhythmias

A

C. Assess need for cardiac transplantation

Patients with a peak oxygen uptake (vo2 <14 mL/kg per min have been shown, in general, to have beer survival when transplanted than when treated medically

567
Q

Which of the following is the most important component of acute management for a typical type of acute decompensated heart failure with elevated blood pressure?
A. Noninvasive ventilator support
B. Vasodilator therapy
C. Inotropic therapy
D. Diuretic therapy

A

B. Vasodilator therapy

Typical ADHF with normal BP are usually volume overloaded so will require diuretic therapy while those with HTN are usually NOT volume overloaded thus should be managed with vasodilators

568
Q

Which of the following is a component of management of patients with cardiogenic type of acute decompensated heart failure?
A. Inotropic therapy
B. Vasodilator therapy
C. Anti-arrhythmic therapy
D. Opiates

A

A. Inotropic therapy

§ Inotropic support and mechanical circulatory support § Diuretic therapy for congestion and renal failure

569
Q

A 48-year-old woman consults for easy fatigability. She appears cachectic and has had repeated admissions for pneumonia in the past 2 years. PE ndings are as follows: BP=90/60, HR=80, RR=18, irregularly-irregular rhythm, (+) RV heave, palpable P2, xed-split S2, (+) II/VI midsystolic murmur at the left sternal border, (+) II/VI mid-diastolic rumbling murmur at the 4 ICS left sternal border. What is the most likely diagnosis?
A. Mitral stenosis
B. Atrial septal defect
C. Ventricular septal defect
D. Tricuspid stenosis/Mitral regurgitation

A

B. Atrial septal defect

ASD – functional murmurs due to volume overload

The combination of fixed split S2, murmurs indicative of increased right-sided flow, and clinical features (recurrent pneumonia, cachexia) strongly point to atrial septal defect (ASD)

570
Q

Which of the following statements is true about ventricular septal defect?
A. Clinical manifestations, course and feasibility of repair are dependent on the left ventricular function
B. The defect is most commonly single and located in the membranous or mid-muscular portion of the septum
C. Functional disturbance depends primarily on the location of the defect
D. Diagnosed cases in adults usually have isolated large defects

A

B. The defect is most commonly single and located in the membranous or mid-muscular portion of the septum

Single defect at the membranous or midmuscular portion of the septum
The status of the pulmonary vascular bed is the principal determinant of the clinical course, manifestations, and feasibility of repair
Disease progression unusual if pulmo vascular resistance ≤ 1/3 of systemic value
Cases seen initially during adulthood are only small or moderate-size defects

571
Q

Which of the following is an expected pathophysiologic nding in chronic aortic regurgitation?
A. Late in the course of the disease the forward stroke volume increases with consequent increase in LV ejection fraction
B. Increase in stroke volume brought about by concentric hypertrophied LV
C. Increase in the left ventricular end-diastolic volume D. Reduced total stroke volume

A

C. Increase in the left ventricular end-diastolic volume

Major hemodynamic compensation in chronic AR: increased in LVEDV

572
Q

In estimating the ASCVD risk, which of the following is considered as among the pertinent information required for scoring?
A. Intake of aspirin
B. Triglyceride level
C. LDL-Cholesterol level
D. Systolic blood pressure

A

D. Systolic blood pressure

§ Other pertinent information: Nine pertinent information required: gender, HDL-C level, total Cholesterol level, race (white, African-American, others), DM diagnosis, therapy for HTN, SBP, smoking status (smoker, non-smoker)

573
Q

Which of the following clinical data of a patient can be considered an “optimal risk factor” in determining ASCVD risk?
A. LDL-C =170mg/dL
B. BP = 110/70
C. HR = 78 beats/min
D. HDL-C = 45mg/dL

A

B. BP = 110/70

Optimal risk factors:
§ Total cholesterol ≤ 170 mg/dL
§ HDL-cholesterol ≥ 50 mg/dL
§ Systolic BP ≤ 110 mm Hg
§ Not taking medications for hypertension
§ Not a diabetic
§ Not a smoker

574
Q

Based on the 2020 Philippine CPG for dyslipidemia, which of the following individuals without ASCVD and DM, would you start on statin therapy to prevent cardiovascular events?
A. 40-year-old man, hypertensive, with LDL-C 140mg/dL B. 45-year-old woman, pre-menopausal, non-hypertensive, with LDL-C 135mg/dL
C. 46-year-old man, hypertensive, with LDL-C 135mg/dL D. 60-year-old woman, non hypertensive, BMI 24 kg/m2, with LDL-C 140mg/dL

A

C. 46-year-old man, hypertensive, with LDL-C 135mg/dL

Recommend statins among Non-DM and without ASCVD:
if age ≥ 45 with LDL-C ≥ 130mg/dL + ≥ 2 risk factors
Risk factors:
§ Male sex
§ Postmenopausal women
§ Smoker
§ Hypertension
§ BMI >25 kg/m2
§ Family history of premature CHD
§ Proteinuria
§ Left ventricular hypertrophy

575
Q

Based on the 2020 Philippine CPG for dyslipidemia, among patients with DM, an LDL-C goal of < 70mg/dL should be set for which of the following patients?
A. 40-year-old man with HbA1c 7.5 %
B. 50-year-old menopausal woman with BMI 27 kg/m2 C. 40-year-old premenopausal woman with BMI 24.5 kg/m2
D. 50-year-old man with HDL-C 49mg/dL

A

B. 50-year-old menopausal woman with BMI 27 kg/m2

Post menopausal (1) + BMI (1) =2

(DM + >1 risk factor) – not consistent with table indicating that ³ 1 risk factor in patients with DM

576
Q

On PE of a 55-year-old woman admied for dyspnea, you noticed that her jugular venous pressure increased by about 2cms H O with inspiration. Which of the following is an important dierential?
A. Massive pulmonary embolism
B. Dilated cardiomyopathy
C. Cardiac tamponade
D. Left atrial myxoma

A

A. Massive pulmonary embolism

ü Kussmaul sign: rise or failure of JVP to fall with inspiration
ü Classic finding in constrictive pericarditis
Other differentials: restrictive CMP, massive PTE, RV infarction, advanced LV systolic heart failure, post-cardiac surgery

577
Q

Among patients presenting with acute chest discomfort, which accompanying features confers the highest likelihood of AMI?
A. Diaphoresis
B. Associated with exertion
C. Radiation to the right arm or shoulder
D. Nausea and vomiting

A

C. Radiation to the right arm or shoulder

578
Q

A 56-year-old man develops acute chest pain and shortness of breath on his 3 day of admission for inferior wall AMI. BP was 80/60 with HR of 108 and RR of 24/min. A new murmur was appreciated on auscultation. What is the more likely description of this new-onset murmur?
A. Early diastolic
B. Mid diastolic
C. Early systolic
D. Holosystolic

A

C. Early systolic

papillary muscle rupture complication of AMI (develops in inferior, posterior and lateral wall AMI) Presentation: acute severe MR heard as early decrescendo systolic murmur (regurgitation into a normal-sized relatively noncompliant left atrium progressive aenuation of the pressure gradient between the left ventricle and the left atrium during systole owing to the rapid rise in left atrial pressure caused by the sudden volume load into an unprepared, noncompliant chamber)

579
Q

Which of the following types of cardiac murmurs will NOT require further evaluation with transthoracic 2D echo?
A. Grade 2 early systolic murmur in a 30-year-old woman
B. Grade 2 middiastolic murmur in a 20-year-old
C. Continuous murmur that disappears when the patient turns head towards examiner
D. Continuous murmur in a 3-month pregnant woman heard with the diaphragm of a stethoscope applied with a rm pressure

A

D. Continuous murmur in a 3-month pregnant woman heard with the diaphragm of a stethoscope applied with a rm pressure

580
Q

The presence of abdominojugular reflux indicates that the pulmonary artery wedge pressure is more than:
A. 5 mmHg
B. 10 mmHg
C. 15 mmHg
D. 20 mmHg

A

C. 15 mmHg

§ Abdominojugular reflux is produced with rm and consistent pressure over the upper portion of the abdomen, preferably over the right upperquadrant, for >15 s
§ Positive response: sustained rise of >3 cm in the JVP during the application of rm abdominal pressure

580
Q

A 60-year old diabetic man with persistent chest discomfort consults at your outpatient clinic. On checking his blood pressures, you noticed that his right arm BP is 140/80mmHg while his left arm BP is 120/80mmHg. All of the following should be considered among the differentials, EXCEPT?
A. Aortic coarctation
B. Aortic regurgitation
C. Aortic dissection
D. Inflammatory subclavian artery disease

A

B. Aortic regurgitation

§ Normal BP dierence between arms is <10mmHg
§ Possible causes of higher BP differential: atherosclerotic or inflammatory subclavian artery disease, supravalvular aortic stenosis, aortic coarctation, aortic dissection

all related to aorta. subclavian is a branch of the aorta

581
Q

The only approved pharmacotherapy for CTEPH if surgical pulmonary endarterectomy is ineffective/ contraindicate

A

Riociguat

inc cGMP

582
Q

Sickle cell is under what WHO group of pulmonary hypertension

A

Group 5

SSS (555)

Sickle cell
Schistosomiasis
Sarcoidosis

583
Q

Isolated inc in post capillary pressure > 15 mmHg is seen in WHO group ___ of pulmonary hpn

A

Group2

If post capillary presure >15 and PVR also >3 and mPAP >20 –> combined pre and post capillary PH (WHO group 2 and 5)

584
Q

A 28-year-old woman presents with fever and chills. Cardiac auscultation revealed a leathery sound heard during both systole and diastole. What is true about the case?
A. She will likely have a positive blood culture
B. 12L ECG will show diuse T segment elevation
C. An increase in the volume of pericardial eusion intensies the sound
D. This is highly sensitive PE nding of the likely diagnosis

A

B. 12L ECG will show diuse T segment elevation

Diagnosis: acute pericarditis
PE: friction rub (highly specic for acute pericarditis, but not sensitive)

585
Q

xA 65-year-old woman who ambulates with support is suspected to have CAD. Which of the following is preferred for initial diagnosis of CAD in this patient?
A. Dobutamine stress myocardial perfusion imaging
B. Exercise stress myocardial perfusion imaging
C. Submaximal exercise myocardial perfusion imaging D. Dipyridamole stress myocardial perfusion imaging

A

D. Dipyridamole stress myocardial perfusion imaging

Exercise stress – generally preferred being physiologic and able to provide additional clinically important information (i.e., clinical and hemodynamic responses, ST-segment changes, exercise duration, and functional status)
Submaximal eort lowers sensitivity of test thus avoided, especially if test is for initial CAD diagnosis

IF unable to exercise or who exercise submaximally, do pharmacologic stress
Most common stressors used: coronary vasodilators (dipyridamole, adenosine, regadenoson) Alternative stressors: b1 agonist (Dobutamine)

586
Q

A CT calcium score of 50 Agatston units is classifed as
A. Minimal
B. Mild
C. Moderate
D. Severe

A

B. Mild

Agatston score:
Minimal: 0-10
Mild: 10-100
Moderate: 100-400
Severe: >400

587
Q

A 58-year-old woman with breast cancer is undergoing adjuvant chemotherapy. Her pre-treatment baseline 2D echo showed an LVEF of 66%. She is asymptomatic. In which scenario would you already consider chemotherapy-induced cardiotoxicity?
A. LVEF of 54%
B. LVEF of 60%
C. LV diastolic dysfunction
D. LV systolic dysfunction

A

A. LVEF of 54%

Suspect cardiotoxicity in the following scenarios: § If with HF symptoms: >5% reduction in LVEF to <55% § If without HF symptoms: >10% reduction in LVEF to <55%

588
Q

A male patient with tachycardia and palpitations has the following ECG tracing with atrial flutter. He was given Adenosine IV. What will be the eect of this intervention?
A. Increase in the AV block
B. Increase in the cardiac rate
C. Termination of the tachycardia
D. No effect

A

A. Increase in the AV block

589
Q

Which of the following is a limitation in the utility of serum BNP in the heart failure?
A. Uncertainty in the diagnosis of an ambulatory patient with dyspnea
B. Prognosticating or assessing severity of chronic heart failure
C. Optimizing medical therapy among euvolemic patients
D. Assessing response of CHF patients to ARNIs

A

D. Assessing response of CHF patients to ARNIs

BNP/ntPRO-BNP can be falsely elevated in the following: § Older age
§ Renal impairment
§ Women
§ Right heart failure (from any cause)
§ Use of ARNIs

Falsely low levels: obese patients

590
Q

Which of the following is a feature of typical acute decompensated HF
A. Hypotension
B. Features of hypoperfusion
C. Presence of biomarkers of injury
D. Pulmonary congestion with hypoxia

A

C. Presence of biomarkers of injury

D = cardiogenic shock

591
Q

Which of the following is a feature of dilated cardiomyopathy?
A. Systolic function is preserved until late in the course of the disease
B. Among patients with new-onset cardiomyopathy, spontaneous recovery is rare
C. Combine beta-blockade and RAAS inhibition may induce dramatic response among those with long-standing disease
D. No identied indication for cardiac resynchronization pacing

A

C. Combine beta-blockade and RAAS inhibition may induce dramatic response among those with long-standing disease

§ Dilated CMP: enlarged LV with depressed systolic function
§ Spontaneous recovery is seen in almost half of new-onset CMP
§ Cardiac resynchronization pacing for those with LBBB preceding clinical heart failure by many years: improves EF and reduces ventricular size

592
Q

Which of the following causes of toxic cardiomyopathy results from generation of reactive oxygen species involving heme compounds?
A. Alcohol
B. Anthracycline
C. Trastuzumab
D. Catecholaminergic stimulant

A

B. Anthracycline

593
Q

A 45-year-old man, diagnosed with ankylosing spondylitis, complains of increased awareness of his heartbeat especially when lying flat on bed. He has visible neck pulsations, with PMI and apex beat displaced to the 5 ICS left mid-axillary line, and a grade IV blowing diastolic murmur at the 3 ICS left parasternal border. What is the primary mechanism of injury of the cardiac lesion?
A. Valve scarring and retraction
B. Widening of the aortic annulus
C. Retrograde dissection of the aorta involving the aortic annulus
D. Cellular inltration and scarring of the media of the thoracic aorta

A

A. Valve scarring and retraction

Valve scarring and retraction may contribute to AR from aortic root disease Widening of the aortic annulus causes: cystic medial necrosis of the ascending aorta (with or without other manifestations of Marfan syndrome), idiopathic dilatation of aorta, osteogenesis imperfecta, severe chronic hypertension, annuloaortic ectasia

594
Q

What drug therapy is useful across all types of the most common valvular heart diseases?
A. Diuretics
B. Beta-blockers
C. ACE-inhibitors
D. Angiotensin receptor blockers

A

A. Diuretics

595
Q

A young man you have been managing for acute idiopathic pericarditis comes back still with chest pain even after being given aspirin and indomethacin. How will you manage the pain?
A. Start Colchicine
B. Start Prednisone
C. Start Anakinra
D. Refer to TCVS for pericardial stripping

A

A. Start Colchicine

Management of acute idiopathic pericarditis
§ First line: aspirin
§ Alternatives: ibuprofen, indomethacin
§ If unresponsive to NSAID: colchicine
§ Prednisone for failed anti-inflammatory therapy but given only for 2-14 days then tapered due to increased risk of recurrence

596
Q

Which is a feature of Buerger’s disease?
A. NOT associated with superficial vein phlebitis
B. Presents with intermittent claudication
C. Higher prevalence among older population
D. Primarily involves medium to large sized arteries and veins

A

B. Presents with intermittent claudication

§ Triad: Raynauds, Claudication, migratory superficial vein phlebitis
§ Aka: thromboangiitis obliterans inflammatory occlusion of medium to small-sized arteries and veins
§ Risks: male <40, Asians, cigaree smoking

597
Q

In a patient with abdominal aortic aneurysm, which of the following symptoms should prompt the physician to consider a risk of imminent rupture?
A. Syncope
B. Lower back pain
C. Bilateral paresthesia
D. Sensation of strong abdominal pulsation

A

B. Lower back pain

Pain: abdomen, chest, lower back or scrotum

§ Harbinger of imminent rupture thus an emergency
§ Acute rupture: severe pain + hypotension

Pathology of abdominal aortic aneurysm: atherosclerosis

Screening for abdominal aortic aneurysm is indicated if § Men 65-75 years who have ever smoked
§ Siblings or ospring of persons with abdominal arterial aneurysm
§ History of thoracic aortic or peripheral arterial aneurysm

598
Q

Based on the 2020 CPG for the management of hypertension in the Philippines, hypertension is based on
A. Ambulatory blood pressure of at least 130/80 on at least two separate days
B. Home blood pressure of at least 140/90 on at least two separate days
C. Office blood pressure of at least 140/90 on at least two separate days
D. Office blood pressure of at least 130/80 on at least two separate days

A

C. Office blood pressure of at least 140/90 on at least two separate days

599
Q

Jasper is a 24 year old vegan who has been experiencing difficulty of breathing. It started 4 months ago when he started to feel tired and exhausted after work. He would be be awaken at night just to catch his breath. Last month he noticed that his ankles were swollen. Physical examination showed pale nailbeds and conjunctivae. He had a grade 2/6 systolic murmur at the 4th ICS LPSB. His cardiologist ordered a 2D echocardiogram and results showed an EF of 45%. What is the cause of the heart failure?
a. Chronic pressure overload
b. Pathologic hypertrophy
c. High output states
d. Chronic volume overload

A

Cause of heart failure: chronic anemia and possible nutritional disorders because patient is vegan.

The correct answer is: High output states

600
Q

Charlie, 59/M, consults for ankle swelling, bilateral for one month now. He is a known hypertensive since 2010 on Losartan 50 mg/tab once daily. Last year, he started to have intermittent chest pains especially when he is stressed, gasps for air every time he would climb the stairs of the LRT station. He was given a medication that reduces systemic vascular resistance and induces arterial vasodilation by his physician. Which of the following medications would have this particular mechanism of action?
a. Beta blocker
b. Aldosterone antagonist
c. Hydralazine
d. Nitroglycerin

A

c. Hydralazine

Hydralazine reduces systemic vascular resistance and induces arterial vasodilation

Nitrates are transformed in smooth muscle cells into nitric oxide which stimulates cyclic GMP and consequent arterial-venous vasodilation

Aldosterone antagonism is associated with a reduction in mortality in all stages of symptomatic NYHA class II to IV HFrEF
* Aldosterone promote sodium imbalance
* Electrolyte imbalance
* Endothelial dysfunction
* Myocardial fibrosis

Beta blockers and RAAS blockers form the cornerstone of pharmacotherapy

601
Q

Dr. Ignacio was observing the treadmill exercise test of Evan, a 54/M, who complains of occasional epigastric pain whenever he goes up a flight of stairs. After 10 minutes, Dr. Ignacio stopped the exercise test. Which of the following may lead to discontinuation of the test?
a. ST elevation 0.1 mV above baseline
b. Fall of systolic blood pressure by 5 mmHg
c. Bradycardia during exercise
d. Severe shortness of breath

A

d. Severe shortness of breath
Indications for discontinuation of Treadmill Test
* chest discomfort
* severe shortness of breath
* dizziness
* severe fatigue
* ST-segment depression >0.2 mV (2 mm)
* a fall in systolic blood pressure >10 mmHg
* the development of a ventricular tachyarrhythmia